Производная для чайников: Производная функции – геометрический смысл и правила дифференцирования

Содержание

Производная функции для чайников — курсы по математике «Geniusmath»

Каждый школьник в старших классах сталкивается с таким понятием, как производная. Кажется, что алгебра, которая и так была не самым простым предметом, полностью теряет связь с реальностью. Так как тема традиционно плохо освещена в школьной программе, мы решили написать краткое руководство по производной функции для чайников.

Внимание! Все определения в данной статье являются лишь интерпретацией строгих научных определений и носят ознакомительный характер. Цель данной статьи — дать базовое понимание такого понятия, как производная функция.

Производная и скорость движения точки

Начнем из далека, и это будет очень важно для понимания! Рассмотрим точку, которая движется по известному нам закону. Если скорость движения непостоянная, то координата точки задается некой функцией  . К примеру, f(5) – координата в момент времени 5, f(0) – координата в начальный момент времени. Поставим задачу: найти скорость движения в каждый момент времени.

Возьмем, для примера, следующий закон движения: .

В начальный момент времени наша координата равна . В момент времени 3.14 (число ): . Построим график зависимости с помощью Excel:

Закон движения точки

Вернемся к нашей задаче. Из курса физики мы знаем, что скорость на заданном промежутке времени можно посчитать, разделив изменение координаты на длину промежутка времени:

Данная формула определяет среднюю скорость на заданном участке . К примеру, скорость нашего движения за первые 3,14 секунды вычисляется следующим образом:

Применим данных подход для нахождения скорости на всем участке движения. Разобьем всё время движения на небольшие промежутки длиной в полсекунды и на каждом из них посчитаем скорость по указанной формуле. Мы получим значения для всех участков. Для наглядности построим график:

Средняя скорость с шагом 0.5 cек

Таким образом мы определили среднюю скорость движения на участках длиной в полсекунды, что практически является решением поставленной задачи. Но что если нам требуется более высокая точность, ведь мы до сих пор ничего не знаем о скорости внутри этих промежутков?  Верно: уменьшим длину промежутков: до 0.1 секунды, затем до 0.01 и так далее.  Будем уменьшать длину до тех пор, пока длина промежутка не будет бесконечно мала (практически 0) и отрезок можно будет принять за точку. В результате мы получим среднюю скорость на бесконечно маленьком промежутке времени, что по сути является

мгновенной скоростью в данный момент времени или просто скоростью в данный момент.

Построим график для полученных значений с помощью Excel: 

Мгновенная скорость

Аналогичным образом считается производная функции, физический смысл которой является мгновенной скоростью изменения функции в данной точке.

Определение производной «на пальцах»

Рассмотрим функцию , определенную на отрезке  и произвольные точки  и  из интервала . Данный треугольник называется “дельта”, он обозначает изменение указанной переменной.

Иными словами, вторая точка находится на неком расстоянии  от первой. Длина отрезка  равна .

Вычислим значения функции в указанных точках и найдем разность: . Мы получим изменение значения функции на отрезке  (в примере выше это изменение координаты точки за некий промежуток времени). Разделив данную разность на длину промежутка, мы получим среднюю скорость изменения функции на данном отрезке. Теперь устремим длину отрезка к бесконечно маленькому значению, как мы это делали в нашем примере, и получим мгновенную скорость изменения функции

Данная величина называется производной функции в точке  и обозначается .

Иными словами, производная функции в точке — это её мгновенная скорость изменения в данной точке.

Если же существует функция , которая в каждой точке  равна производной функции  в данной точке, то говорят, что  — производная функции 

О том, как данные производные считаются, поговорим в статье «Нахождение производной».

Производная произведения и частного функции

Формула производной произведения функции имеет вид .

Формула производной частного функции имеет вид .

Однако было бы наивно надеяться, что на контрольной или экзамене Вам обязательно попадётся пример на нахождение производной такого частного: , где легко подставить простенькое выражение в формулу и выдать правильное решение.

В реальных задачах требуется найти производную таких произведений и частных, в которые вкрались тригонометрические выражения и логарифмы, не говоря уже о множителях (константах), и вообще о том, что может содержать произведение или частное функции. Поэтому примеры нахождения производной произведения и частного функций вынесены в эту отдельную статью.

Пример 1.

Найти производную функции

.

Решение. От нас требуется найти производную произведения функций. Прежде всего вынесем множитель 2 за знак производной:

.

Теперь применяем формулу дифференцирования произведения:

Приводим слагаемые в скобках к общему знаменателю:

В числителе первого слагаемого можно заметить знакомое по школьной математике выражение двойного угла:

Существует также известное из школьной математики тождество:

.

Подставляем его в наш промежуточный результат и получаем:

.

Производная данного произведения найдена.

Пример 2.Найти производную функции

.

Пример 4.Найти производную функции

Решение. Перед нами сумма частных. Следовательно, каждое слагаемое будет дифференцировано как частное. Применяем правило дифференцирования частного, не забывая, чему равны производные числа(константы) и самой переменной x:

Пример 5.

Найти производную функции

Шаг 1. Применим правило дифференцирования частного:

Шаг 2. Находим производную произведения в числителе:

Шаг 3. Находим производную суммы:

Шаг 4. Находим производную функции:

Чтобы избавиться от дроби в числителе, умножаем числитель и знаменатель на x:

Пример 6.Найти производную функции

.

Пример 8.Найти производную функции

Шаг 1. Применим правило дифференцирования произведения:

Шаг 2. Найдём производную частного, помня, что производная константы равна нулю, а корень из константы является также константой:

Шаг 3. Находим производную арктангенса (формула 12 в таблице производных):

Искомая производная:

Проверить решение именно Вашей задачи можно на калькуляторе производных.

Пример 9.Найти производную функции

Шаг 1. Применим правило дифференцирования частного:

Шаг 2. Дифференцируем по правилам для произведения и показательной функции (формула 17 в таблице производных):

Чтобы избавиться от дроби в числителе, умножаем числитель и знаменатель на :

Вновь настоятельно рекомендуем изучить производную сложной функции.

Поделиться с друзьями

Весь блок “Производная”

определение, как найти, примеры решений. Правила вычисления производных

Решать физические задачи или примеры по математике совершенно невозможно без знаний о производной и методах ее вычисления. Производная – одно из важнейших понятий математического анализа. Этой фундаментальной теме мы и решили посвятить сегодняшнюю статью. Что такое производная, каков ее физический и геометрический смысл, как посчитать производную функции? Все эти вопросы можно объединить в один: как понять производную?

Геометрический и физический смысл производной

Пусть есть функция f(x) , заданная в некотором интервале (a, b) . Точки х и х0 принадлежат этому интервалу. При изменении х меняется и сама функция. Изменение аргумента – разность его значений х-х0 . Эта разность записывается как дельта икс и называется приращением аргумента. Изменением или приращением функции называется разность значений функции в двух точках. Определение производной:

Производная функции в точке – предел отношения приращения функции в данной точке к приращению аргумента, когда последнее стремится к нулю.

Иначе это можно записать так:

Какой смысл в нахождении такого предела? А вот какой:

производная от функции в точке равна тангенсу угла между осью OX и касательной к графику функции в данной точке.


Физический смысл производной: производная пути по времени равна скорости прямолинейного движения.

Действительно, еще со школьных времен всем известно, что скорость – это частное пути x=f(t) и времени t . Средняя скорость за некоторый промежуток времени:

Чтобы узнать скорость движения в момент времени t0 нужно вычислить предел:

Правило первое: выносим константу

Константу можно вынести за знак производной. Более того – это нужно делать. При решении примеров по математике возьмите за правило – если можете упростить выражение, обязательно упрощайте .

Пример. Вычислим производную:

Правило второе: производная суммы функций

Производная суммы двух функций равна сумме производных этих функций. То же самое справедливо и для производной разности функций.

Не будем приводить доказательство этой теоремы, а лучше рассмотрим практический пример.

Найти производную функции:

Правило третье: производная произведения функций

Производная произведения двух дифференцируемых функций вычисляется по формуле:

Пример: найти производную функции:

Решение:

Здесь важно сказать о вычислении производных сложных функций. Производная сложной функции равна произведению производной этой функции по промежуточному аргументу на производную промежуточного аргумента по независимой переменной.

В вышеуказанном примере мы встречаем выражение:

В данном случае промежуточный аргумент – 8х в пятой степени. Для того, чтобы вычислить производную такого выражения сначала считаем производную внешней функции по промежуточному аргументу, а потом умножаем на производную непосредственно самого промежуточного аргумента по независимой переменной.

Правило четвертое: производная частного двух функций

Формула для определения производной от частного двух функций:

Мы постарались рассказать о производных для чайников с нуля. Эта тема не так проста, как кажется, поэтому предупреждаем: в примерах часто встречаются ловушки, так что будьте внимательны при вычислении производных.

С любым вопросом по этой и другим темам вы можете обратиться в студенческий сервис . За короткий срок мы поможем решить самую сложную контрольную и разобраться с заданиями, даже если вы никогда раньше не занимались вычислением производных.

Приложение

Решение производной на сайт для закрепления пройденного материала студентами и школьниками. Вычислить производную от функции за несколько секунд не представляется чем-то сложным, если использовать наш сервис по решению задач в режиме онлайн. Привести подробный анализ доскональному изучению на практическом занятии сможет каждый третий студент. Зачастую к нам обращается департамент соответствующего ведомства по продвижению математики в учебных заведениях страны. Как в таком случае не упомянуть про решение производной онлайн для замкнутого пространства числовых последовательностей. Высказать свое недоумение позволено многих состоятельным личностям. Но между делом математики не сидят на месте и много работают. Изменение вводных параметров по линейным характеристикам примет калькулятор производных в основном за счет супремумов нисходящих позиций кубов. Итог неизбежен как поверхность. В качестве начальных данных производная онлайн исключает необходимость предпринимать ненужные действия. За исключением вымышленных домашних работ. Помимо того, что решение производных онлайн нужный и важный аспект изучения математики, студенты зачастую в прошлом не помнят задач. Студент, как ленивое существо, это понимает. Но студенты – веселые люди! Либо делать по правилам, либо производная функции в наклонной плоскости может придать ускорение материальной точке. Куда-то направим вектор нисходящего пространственного луча. В нужном ответе найти производную кажется абстрактным теоретическим направлением из-за неустойчивости математической системы. Задумаем отношение чисел как последовательность неиспользуемых вариантов. Канал связи пополнился пятой линий по вектору убывания из точки замкнутого раздвоения куба. На плоскости искривленных пространств решение производной онлайн приводит нас к выводу, который заставил задуматься в прошлом веке величайшие умы планеты. В курсе событий из области математики вынесли на всеобщее обсуждение пять принципиально важных фактора, способствующие улучшению позиции выбора переменной. Вот и закон для точек гласит, что производная онлайн подробно вычисляется не в каждом случае, исключением может быть только лояльно прогрессирующий момент. Прогноз вывел нас на новый виток развития. Нужен результат. В линию прошедшего под поверхность математического наклона калькулятор производных режима находятся в области пересечения произведений на множестве изгиба. Осталось проанализировать дифференцирование функции в её независимой точке около эпсилон-окрестности. В этом можно убедиться каждому на практике. В итоге будет что решать на следующем этапе программирования. Студенту производная онлайн нужна как всегда независимо от практикуемых воображаемых исследований. Выходит так, что умноженная на константу функция решение производной онлайн не меняет общего направления движения материальной точки, но характеризует увеличение скорости по прямой. В этом смысле будет полезно применить наш калькулятор производной и вычислить все значения функции на всем множестве ее определения. Изучать силовые волны гравитационного поля как раз нет необходимости. Ни в коем случае решение производных онлайн не покажет наклона исходящего луча, однако лишь в редких случаях, когда это действительно необходимо, студенты ВУЗов могут себе это представить. Исследуем принципала. Значение наименьшего ротора прогнозируемо. Применить к результату смотрящих направо линий, по которым описывается шар, но онлайн калькулятор производных это есть основа для фигур особой прочности и нелинейной зависимости. Отчет по проекту математики готов. Личные характеристики разность наименьших чисел и производная функции по оси ординат выведет на высоту вогнутость той же функции. Есть направление – есть вывод. Легче выдвинуть теорию на практике. Есть предложение у студентов по срокам начала исследования. Нужен преподавателя ответ. Снова, как и к предыдущему положению, математическая система не регулируема на основании действия, которое поможет найти производную. Как и нижний полулинейный вариант производная онлайн подробно укажет на выявленность решения по вырожденному условному закону. Как раз выдвинута идея по расчету формул. Линейное дифференцирование функции отклоняет истинность решения на простое выкладывание неуместных положительных вариаций. Важность знаков сравнения будет расценена как сплошной разрыв функции по оси. В том заключается важность самого осознанного вывода, по мнению студента, при котором производная онлайн есть нечто иное, чем лояльный пример мат анализа. Радиус искривленного круга в пространстве Евклидовом напротив дал калькулятор производных естественному представлению обмена решительных задач на устойчивость. Лучший метод найден. Было проще ставить задание на уровень вверх. Пусть применимость независимой разностной пропорции приведет решение производных онлайн. Крутится решение вокруг оси абсцисс, описывая фигуру круга. Выход есть, и он основан на теоретически подкрепленных студентами ВУЗов исследованиях, по которым учится каждый, и даже в те моменты времени существует производная функции. Нашли прогрессу дорогу и студенты подтвердили. Мы можем позволить себе найти производную, не выходя за рамки неестественного подхода в преобразовании математической системы. Левый знак пропорциональности растет с геометрической последовательностью как математическое представление онлайн калькулятора производных за счет неизвестного обстоятельства линейных множителей на бесконечной оси ординат. Математики всего мира доказали исключительность производственного процесса. Есть наименьший квадрат внутри круга по описанию теории. Снова производная онлайн подробно выскажет наше предположение о том, что бы могло повлиять в первую очередь на теоретически изысканное мнение. Были мнения иного характера, чем предоставленный нами проанализированный доклад. Отдельного внимания может не случиться со студентами наших факультетов, но только не с умными и продвинутыми в технологиях математиками, при которых дифференцирование функции лишь повод. Механический смысл производной очень прост. Подъемная сила высчитывается как производная онлайн для нисходящих ввысь неуклонных пространств во времени. Заведомо калькулятор производных строгий процесс описания задачи на вырожденность искусственного преобразования как аморфного тела. Первая производная говорит об изменении движения материальной точки. Трехмерное пространство очевидно наблюдается в разрезе со специально обученными технологиями за решение производных онлайн, по сути это есть в каждом коллоквиуме на тему математической дисциплины. Вторая производная характеризует изменение скорости материальной точки и определяет ускорение. Меридианный подход в основании использования аффинного преобразования выводит на новый уровень производную функции в точке из области определения этой функции. Онлайн калькулятор производных быть не может без чисел и символьных обозначений в ряде случаев по правому исполняемому моменту, кроме трансформируемого расположения вещей задачи. Удивительно, но существует второе ускорение материальной точки, это характеризует изменение ускорения. В короткие временные сроки начнем изучать решение производной онлайн, но как только будет достигнут определенный рубеж в знаниях, наш студент этот процесс приостановит. Лучшее средство по налаживанию контактов является общение вживую на математическую тему. Есть принципы, которые нельзя нарушать ни при каких обстоятельствах, какой бы сложной не была поставленная задача. Полезно найти производную онлайн вовремя и без ошибок. Приведет это к новому положению математического выражения. Система устойчива. Физический смысл производной не так популярен, как механический. Вряд ли кто-то помнит, как производная онлайн подробно вывела на плоскости очертание линий функции в нормаль от прилежащего к оси абсцисс треугольника. Большую роль в исследованиях прошлого века заслуживает человек. Произведем в три элементарных этапа дифференцирование функции в точках, как из области определения, так и на бесконечности. Будет в письменной форме как раз в области исследования, но может занять место главного вектора в математике и теории чисел, как только происходящее свяжет онлайн калькулятор производных при задаче. Была бы причина, а повод составить уравнение будет. Очень важно иметь в виду все входные параметры. Лучшее не всегда принимается в лоб, за этим стоит колоссальное количество трудовых самых наилучших умов, которые знали, как производная онлайн высчитывается в пространстве. С тех пор выпуклость считается свойством непрерывной функции. Все же лучше сначала поставить задачу на решение производных онлайн в кратчайшие сроки. Таким образом, решение будет полным. Кроме невыполненных норм это не считается достаточным. Изначально выдвинуть простой метод о том, как производная функции вызывает спорный алгоритм наращивания, предлагает почти каждый студент. По направлению восходящего луча. В этом есть смысл как в общем положении. Ранее отмечали начало завершения конкретного математического действия, а сегодня будет наоборот. Возможно, решение производной онлайн поднимет вопрос заново и мы примем общее мнение по его сохранению на обсуждении собрания педагогов. Надеемся на понимание со всех сторон участниц собрания. Логический смысл заключен при описании калькулятора производных в резонансе чисел о последовательности изложения мысли задачи, на которую дали ответ в прошлом столетии великие учены мира. Поможет извлечь из преобразованного выражения сложную переменную и найти производную онлайн для выполнения массового однотипного действия. Истина в разы лучше догадок. Наименьшее значение в тренде. Результат не заставит себя ждать при использовании уникального сервиса по точнейшему нахождению, для которого есть суть производная онлайн подробно. Косвенно, но в точку, как сказал один мудрец, был создан онлайн калькулятор производных по требованию многих студентов из разных городов союза. Если разница есть, то зачем решать дважды. Заданный вектор лежит по одну сторону с нормалью. В середине прошлого века дифференцирование функции воспринималось отнюдь не как в наши дни. Благодаря развитию в прогрессе, появилась математика онлайн. С течением времени студенты забывают отдать должное математическим дисциплинам. Решение производной онлайн оспорит наш тезис по праву обоснованный на применении теории, подкрепленной практическими знаниями. Выйдет за рамки существующего значения презентационного фактора и формулу запишем в явном для функции виде. Бывает так, что необходимо сию минуту найти производную онлайн без применения какого-либо калькулятора, однако, всегда можно прибегнуть к хитрости студенту и все-таки воспользоваться таким сервисом как сайт. Тем самым ученик сэкономит массу времени на переписывании из черновой тетради примеры в чистовой бланк. Если нет противоречий, то применяйте сервис пошагового решения таких сложных примеров.

На этом занятии мы будем учиться применять формулы и правила дифференцирования.

Примеры. Найти производные функций.

1. y=x 7 +x 5 -x 4 +x 3 -x 2 +x-9. Применяем правило I , формулы 4, 2 и 1 . Получаем:

y’=7x 6 +5x 4 -4x 3 +3x 2 -2x+1.

2. y=3x 6 -2x+5. Решаем аналогично, используя те же формулы и формулу 3.

y’=3∙6x 5 -2=18x 5 -2.

Применяем правило I , формулы 3, 5 и 6 и 1.

Применяем правило IV , формулы 5 и 1 .

В пятом примере по правилу I производная суммы равна сумме производных, а производную 1-го слагаемого мы только что находили (пример 4 ), поэтому, будем находить производные 2-го и 3-го слагаемых, а для 1-го слагаемого можем сразу писать результат.

Дифференцируем 2-ое и 3-е слагаемые по формуле 4 . Для этого преобразуем корни третьей и четвертой степеней в знаменателях к степеням с отрицательными показателями, а затем, по 4 формуле, находим производные степеней.

Посмотрите на данный пример и полученный результат. Уловили закономерность? Хорошо. Это означает, что мы получили новую формулу и можем добавить ее в нашу таблицу производных.

Решим шестой пример и выведем еще одну формулу.

Используем правило IV и формулу 4 . Получившиеся дроби сократим.

Смотрим на данную функцию и на ее производную. Вы, конечно, поняли закономерность и готовы назвать формулу:

Учим новые формулы!

Примеры.

1. Найти приращение аргумента и приращение функции y=x 2 , если начальное значение аргумента было равно 4 , а новое –4,01 .

Решение.

Новое значение аргумента х=х 0 +Δx . Подставим данные: 4,01=4+Δх, отсюда приращение аргумента Δх =4,01-4=0,01. Приращение функции, по определению, равно разности между новым и прежним значениями функции, т.е. Δy=f (х 0 +Δх) – f (x 0). Так как у нас функция y=x 2 , то Δу =(х 0 +Δx) 2 — (х 0) 2 =(х 0) 2 +2x 0 · Δx+(Δx) 2 — (х 0) 2 =2x 0 · Δx+(Δx) 2 =

2 · 4 · 0,01+(0,01) 2 =0,08+0,0001=0,0801.

Ответ: приращение аргумента Δх =0,01; приращение функции Δу =0,0801.

Можно было приращение функции найти по-другому: Δy =y (х 0 +Δx) -y (х 0)=у(4,01) -у(4)=4,01 2 -4 2 =16,0801-16=0,0801.

2. Найти угол наклона касательной к графику функции y=f (x) в точке х 0 , если f “(х 0) = 1 .

Решение.

Значение производной в точке касания х 0 и есть значение тангенса угла наклона касательной (геометрический смысл производной). Имеем: f “(х 0) = tgα = 1 → α = 45°, так как tg45°=1.

Ответ: касательная к графику данной функции образует с положительным направлением оси Ох угол, равный 45° .

3. Вывести формулу производной функции y=x n .

Дифференцирование — это действие нахождения производной функции.

При нахождении производных применяют формулы, которые были выведены на основании определения производной, так же, как мы вывели формулу производной степени: (x n)” = nx n-1 .

Вот эти формулы.

Таблицу производных легче будет заучить, проговаривая словесные формулировки:

1. Производная постоянной величины равна нулю.

2. Икс штрих равен единице.

3. Постоянный множитель можно вынести за знак производной.

4. Производная степени равна произведению показателя этой степени на степень с тем же основанием, но показателем на единицу меньше.

5. Производная корня равна единице, деленной на два таких же корня.

6. Производная единицы, деленной на икс равна минус единице, деленной на икс в квадрате.

7. Производная синуса равна косинусу.

8. Производная косинуса равна минус синусу.

9. Производная тангенса равна единице, деленной на квадрат косинуса.

10. Производная котангенса равна минус единице, деленной на квадрат синуса.

Учим правила дифференцирования .

1. Производная алгебраической суммы равна алгебраической сумме производных слагаемых.

2. Производная произведения равна произведению производной первого множителя на второй плюс произведение первого множителя на производную второго.

3. Производная «у», деленного на «вэ» равна дроби, в числителе которой “у штрих умноженный на «вэ» минус «у, умноженный на вэ штрих», а в знаменателе — «вэ в квадрате».

4. Частный случай формулы 3.

Учим вместе!

Страница 1 из 1 1

Задача нахождения производной от заданной функции является одной из основных в курсе математики старшей школы и в высших учебных заведениях. Невозможно полноценно исследовать функцию, построить ее график без взятия ее производной. Производную функции легко можно найти, зная основные правила дифференцирования, а также таблицу производных основных функций. Давайте разберемся, как найти производную функции.

Производной функции называют предел отношения приращения функции к приращению аргумента, когда приращение аргумента стремится к нулю.

Понять это определение достаточно сложно, так как понятие предела в полной мере не изучается в школе. Но для того, чтобы находить производные различных функций, понимать определение не обязательно, оставим его специалистам математикам и перейдем сразу к нахождению производной.

Процесс нахождения производной называется дифференцированием. При дифференцировании функции мы будем получать новую функцию.

Для их обозначения будем использовать латинские буквы f, g и др.

Существует много всевозможных обозначений производных. Мы будем использовать штрих. Например запись g” означает, что мы будем находить производную функции g.

Таблица производных

Для того чтобы дать ответ на вопрос как найти производную, необходимо привести таблицу производных основных функций. Для вычисления производных элементарных функций не обязательно производить сложные вычисления. Достаточно просто посмотреть ее значение в таблице производных.

  1. (sin x)”=cos x
  2. (cos x)”= –sin x
  3. (x n)”=n x n-1
  4. (e x)”=e x
  5. (ln x)”=1/x
  6. (a x)”=a x ln a
  7. (log a x)”=1/x ln a
  8. (tg x)”=1/cos 2 x
  9. (ctg x)”= – 1/sin 2 x
  10. (arcsin x)”= 1/√(1-x 2)
  11. (arccos x)”= – 1/√(1-x 2)
  12. (arctg x)”= 1/(1+x 2)
  13. (arcctg x)”= – 1/(1+x 2)
Пример 1. Найдите производную функции y=500.

Мы видим, что это константа. По таблице производных известно, что производная константы, равна нулю (формула 1).

Пример 2. Найдите производную функции y=x 100 .

Это степенная функция в показателе которой 100 и чтобы найти ее производную нужно умножить функцию на показатель и понизить на 1 (формула 3).

(x 100)”=100 x 99

Пример 3. Найдите производную функции y=5 x

Это показательная функция, вычислим ее производную по формуле 4.

Пример 4. Найдите производную функции y= log 4 x

Производную логарифма найдем по формуле 7.

(log 4 x)”=1/x ln 4

Правила дифференцирования

Давайте теперь разберемся, как находить производную функции, если ее нет в таблице. Большинство исследуемых функций, не являются элементарными, а представляют собой комбинации элементарных функций с помощью простейших операций (сложение, вычитание, умножение, деление, а также умножение на число). Для нахождения их производных необходимо знать правила дифференцирования. Далее буквами f и g обозначены функции, а С – константа.

1. Постоянный коэффициент можно выносить за знак производной

Пример 5. Найдите производную функции y= 6*x 8

Выносим постоянный коэффициент 6 и дифференцируем только x 4 . Это степенная функция, производную которой находим по формуле 3 таблицы производных.

(6*x 8)” = 6*(x 8)”=6*8*x 7 =48* x 7

2. Производная суммы равна сумме производных

(f + g)”=f” + g”

Пример 6. Найдите производную функции y= x 100 +sin x

Функция представляет собой сумму двух функций, производные которых мы можем найти по таблице. Так как (x 100)”=100 x 99 и (sin x)”=cos x. Производная суммы будет равна сумме данных производных:

(x 100 +sin x)”= 100 x 99 +cos x

3. Производная разности равна разности производных

(f – g)”=f” – g”

Пример 7. Найдите производную функции y= x 100 – cos x

Эта функция представляет собой разность двух функции, производные которых мы также можем найти по таблице. Тогда производная разности равна разности производных и не забудем поменять знак, так как (cos x)”= – sin x.

(x 100 – cos x)”= 100 x 99 + sin x

Пример 8. Найдите производную функции y=e x +tg x– x 2 .

В этой функции есть и сумма и разность, найдем производные от каждого слагаемого:

(e x)”=e x , (tg x)”=1/cos 2 x, (x 2)”=2 x. Тогда производная исходной функции равна:

(e x +tg x– x 2)”= e x +1/cos 2 x –2 x

4. Производная произведения

(f * g)”=f” * g + f * g”

Пример 9. Найдите производную функции y= cos x *e x

Для этого сначала найдем производного каждого множителя (cos x)”=–sin x и (e x)”=e x . Теперь подставим все в формулу произведения. Производную первой функции умножим на вторую и прибавим произведение первой функции на производную второй.

(cos x* e x)”= e x cos x – e x *sin x

5. Производная частного

(f / g)”= f” * g – f * g”/ g 2

Пример 10. Найдите производную функции y= x 50 /sin x

Чтобы найти производную частного, сначала найдем производную числителя и знаменателя отдельно: (x 50)”=50 x 49 и (sin x)”= cos x. Подставив в формулу производной частного получим:

(x 50 /sin x)”= 50x 49 *sin x – x 50 *cos x/sin 2 x

Производная сложной функции

Сложная функция – это функция, представленная композицией нескольких функций. Для нахождения производной сложной функции также существует правило:

(u (v))”=u”(v)*v”

Давайте разберемся как находить производную такой функции. Пусть y= u(v(x)) – сложная функция. Функцию u назовем внешней, а v – внутренней.

Например:

y=sin (x 3) – сложная функция.

Тогда y=sin(t) – внешняя функция

t=x 3 – внутренняя.

Давайте попробуем вычислить производную этой функции. По формуле необходимо перемножить производные внутренней и внешней функции.

(sin t)”=cos (t) – производная внешней функции (где t=x 3)

(x 3)”=3x 2 – производная внутренней функции

Тогда (sin (x 3))”= cos (x 3)* 3x 2 – производная сложной функции.

Операция отыскания производной называется дифференцированием.

В результате решения задач об отыскании производных у самых простых (и не очень простых) функций по определению производной как предела отношения приращения к приращению аргумента появились таблица производных и точно определённые правила дифференцирования. Первыми на ниве нахождения производных потрудились Исаак Ньютон (1643-1727) и Готфрид Вильгельм Лейбниц (1646-1716).

Поэтому в наше время, чтобы найти производную любой функции, не надо вычислять упомянутый выше предел отношения приращения функции к приращению аргумента, а нужно лишь воспользоваться таблицей производных и правилами дифференцирования. Для нахождения производной подходит следующий алгоритм.

Чтобы найти производную , надо выражение под знаком штриха разобрать на составляющие простые функции и определить, какими действиями (произведение, сумма, частное) связаны эти функции. Далее производные элементарных функций находим в таблице производных, а формулы производных произведения, суммы и частного – в правилах дифференцирования. Таблица производных и правила дифференцирования даны после первых двух примеров.

Пример 1. Найти производную функции

Решение. Из правил дифференцирования выясняем, что производная суммы функций есть сумма производных функций, т. е.

Из таблицы производных выясняем, что производная “икса” равна единице, а производная синуса – косинусу. Подставляем эти значения в сумму производных и находим требуемую условием задачи производную:

Пример 2. Найти производную функции

Решение. Дифференцируем как производную суммы, в которой второе слагаемое с постоянным множителем, его можно вынести за знак производной:

Если пока возникают вопросы, откуда что берётся, они, как правило, проясняются после ознакомления с таблицей производных и простейшими правилами дифференцирования. К ним мы и переходим прямо сейчас.

Таблица производных простых функций

Правила дифференцирования

1. Производная суммы или разности
2. Производная произведения
2a. Производная выражения, умноженного на постоянный множитель
3. Производная частного
4. Производная сложной функции

Правило 1. Если функции

дифференцируемы в некоторой точке , то в той же точке дифференцируемы и функции

причём

т.е. производная алгебраической суммы функций равна алгебраической сумме производных этих функций.

Следствие. Если две дифференцируемые функции отличаются на постоянное слагаемое, то их производные равны , т.е.

Правило 2. Если функции

дифференцируемы в некоторой точке , то в то же точке дифференцируемо и их произведение

причём

т.е. производная произведения двух функций равна сумме произведений каждой из этих функций на производную другой.

Следствие 1. Постоянный множитель можно выносить за знак производной :

Следствие 2. Производная произведения нескольких дифференцируемых функций равна сумме произведений производной каждого из сомножителей на все остальные.

Например, для трёх множителей:

Правило 3. Если функции

дифференцируемы в некоторой точке и , то в этой точке дифференцируемо и их частное u/v , причём

т.е. производная частного двух функций равна дроби, числитель которой есть разность произведений знаменателя на производную числителя и числителя на производную знаменателя, а знаменатель есть квадрат прежнего числителя.

Где что искать на других страницах

При нахождении производной произведения и частного в реальных задачах всегда требуется применять сразу несколько правил дифференцирования, поэтому больше примеров на эти производные – в статье “Производная произведения и частного функций ” .

Замечание. Следует не путать константу (то есть, число) как слагаемое в сумме и как постоянный множитель! В случае слагаемого её производная равна нулю, а в случае постоянного множителя она выносится за знак производных. Это типичная ошибка, которая встречается на начальном этапе изучения производных, но по мере решения уже нескольких одно- двухсоставных примеров средний студент этой ошибки уже не делает.

А если при дифференцировании произведения или частного у вас появилось слагаемое u v , в котором u – число, например, 2 или 5, то есть константа, то производная этого числа будет равна нулю и, следовательно, всё слагаемое будет равно нулю (такой случай разобран в примере 10).

Другая частая ошибка – механическое решение производной сложной функции как производной простой функции. Поэтому производной сложной функции посвящена отдельная статья. Но сначала будем учиться находить производные простых функций.

По ходу не обойтись без преобразований выражений. Для этого может потребоваться открыть в новых окнах пособия Действия со степенями и корнями и Действия с дробями .

Если Вы ищете решения производных дробей со степенями и корнями, то есть, когда функция имеет вид вроде , то следуйте на занятие “Производная суммы дробей со степенями и корнями “.

Если же перед Вами задача вроде , то Вам на занятие “Производные простых тригонометрических функций”.

Пошаговые примеры – как найти производную

Пример 3. Найти производную функции

Решение. Определяем части выражения функции: всё выражение представляет произведение, а его сомножители – суммы, во второй из которых одно из слагаемых содержит постоянный множитель. Применяем правило дифференцирования произведения: производная произведения двух функций равна сумме произведений каждой из этих функций на производную другой:

Далее применяем правило дифференцирования суммы: производная алгебраической суммы функций равна алгебраической сумме производных этих функций. В нашем случае в каждой сумме второе слагаемое со знаком минус. В каждой сумме видим и независимую переменную, производная которой равна единице, и константу (число), производная которой равна нулю. Итак, “икс” у нас превращается в единицу, а минус 5 – в ноль. Во втором выражении “икс” умножен на 2, так что двойку умножаем на ту же единицу как производную “икса”. Получаем следующие значения производных:

Подставляем найденные производные в сумму произведений и получаем требуемую условием задачи производную всей функции:

Пример 4. Найти производную функции

Решение. От нас требуется найти производную частного. Применяем формулу дифференцирования частного: производная частного двух функций равна дроби, числитель которой есть разность произведений знаменателя на производную числителя и числителя на производную знаменателя, а знаменатель есть квадрат прежнего числителя. Получаем:

Производную сомножителей в числителе мы уже нашли в примере 2. Не забудем также, что произведение, являющееся вторым сомножителем в числителе в текущем примере берётся со знаком минус:

Если Вы ищете решения таких задач, в которых надо найти производную функции, где сплошное нагромождение корней и степеней, как, например, , то добро пожаловать на занятие “Производная суммы дробей со степенями и корнями” .

Если же Вам нужно узнать больше о производных синусов, косинусов, тангенсов и других тригонометрических функций, то есть, когда функция имеет вид вроде , то Вам на урок “Производные простых тригонометрических функций” .

Пример 5. Найти производную функции

Решение. В данной функции видим произведение, один из сомножителей которых – квадратный корень из независимой переменной, с производной которого мы ознакомились в таблице производных. По правилу дифференцирования произведения и табличному значению производной квадратного корня получаем:

Пример 6. Найти производную функции

Решение. В данной функции видим частное, делимое которого – квадратный корень из независимой переменной. По правилу дифференцирования частного, которое мы повторили и применили в примере 4, и табличному значению производной квадратного корня получаем:

Чтобы избавиться от дроби в числителе, умножаем числитель и знаменатель на .

определение, как найти, примеры решений

Производная – главнейшее понятие математического анализа. Она характеризует изменение функции аргумента x в некоторой точке. При этом и сама производная является функцией от аргумента x

Производной функции в точке называется предел (если он существует и конечен) отношения приращения функции к приращению аргумента при условии, что последнее стремится к нулю.

Наиболее употребительны следующие обозначения производной :

Пример 1. Пользуясь определением производной , найти производную функции

Решение. Из определения производной вытекает следующая схема её вычисления.

Дадим аргументу приращение (дельта) и найдём приращение функции:

Найдём отношение приращения функции к приращению аргумента:

Вычислим предел этого отношения при условии, что приращение аргумента стремится к нулю, то есть требуемую в условии задачи производную:

Физический смысл производной

К понятию производной привело изучение Галилео Галилеем закона свободного падения тел, а в более широком смысле – задачи о мгновенной скорости неравномерного прямолинейного движения точки.

Пусть камешек поднят и затем из состояния покоя отпущен. Путь s , проходимый за время t , является функцией времени, то есть. s = s (t ). Если задан закон движения точки, то можно определить среднюю скорость за любой промежуток времени. Пусть в момент времени камешек находился в положении A , а в момент – в положении B . За промежуток времени (от t до ) точка прошла путь . Поэтому средняя скорость движения за этот промежуток времени, которую обзначим через , составляет

.

Однако движение свободно падающего тела явно неравномерное. Скорость v падения постоянно возрастает. И средней скорости уже недостаточно для характеристики быстроты движения на различных участках пути. Такая характеристика тем точнее, чем меньше промежуток времени . Поэтому вводится следующее понятие: мгновенной скоростью прямолинейного движения (или скоростью в данный момент времени t ) называется предел средней скорости при :

(при условии, что этот предел существует и конечен).

Так выясняется, что мгновенная скорость есть предел отношения приращения функции s (t ) к приращению аргумента t при Это и есть производная, которая в общем виде записывается так:.

.

Решение обозначенной задачи представляет собой физический смысл производной . Итак, производной функции y=f (x ) в точке x называется предел (если он существует и конечен) приращения функции к приращению аргумента при условии, что последнее стремится к нулю.

Пример 2. Найти производную функции

Решение. Из определения производной вытекает следующая схема для её вычисления.

Шаг 1. Дадим аргументу приращение и найдём

Шаг 2. Найдём приращение функции:

Шаг 3. Найдём отношение приращения функции к приращению аргумента:

Шаг 4. Вычислим предел этого отношения при , то есть производную:

Геометрический смысл производной

Пусть функция определена на интервале и пусть точка М на графике функции соответствует значению аргумента , а точка Р – значению . Проведём через точки М и Р прямую и назовём её секущей . Обозначим через угол между секущей и осью . Очевидно, что этот угол зависит от .

Если существует

проходящую через точку , называют предельным положением секущей МР при (или при ).

Касательной к графику функции в точке М называется предельное положение секущей МР при , или, что то же при .

Из определения следует, что для существования касательной достаточно, чтобы существовал предел

,

причём предел равен углу наклона касательной к оси .

Теперь дадим точное определение касательной.

Касательной к графику функции в точке называется прямая, проходящая через точку и имеющая угловой коэффициент , т.е. прямая, уравнение которой

Из этого определения следует, что производная функции равна угловому коэффициенту касательной к графику этой функции в точке с абсциссой x . В этом состоит геометрический смысл производной.

На этом занятии мы будем учиться применять формулы и правила дифференцирования.

Примеры. Найти производные функций.

1. y=x 7 +x 5 -x 4 +x 3 -x 2 +x-9. Применяем правило I , формулы 4, 2 и 1 . Получаем:

y’=7x 6 +5x 4 -4x 3 +3x 2 -2x+1.

2. y=3x 6 -2x+5. Решаем аналогично, используя те же формулы и формулу 3.

y’=3∙6x 5 -2=18x 5 -2.

Применяем правило I , формулы 3, 5 и 6 и 1.

Применяем правило IV , формулы 5 и 1 .

В пятом примере по правилу I производная суммы равна сумме производных, а производную 1-го слагаемого мы только что находили (пример 4 ), поэтому, будем находить производные 2-го и 3-го слагаемых, а для 1-го слагаемого можем сразу писать результат.

Дифференцируем 2-ое и 3-е слагаемые по формуле 4 . Для этого преобразуем корни третьей и четвертой степеней в знаменателях к степеням с отрицательными показателями, а затем, по 4 формуле, находим производные степеней.

Посмотрите на данный пример и полученный результат. Уловили закономерность? Хорошо. Это означает, что мы получили новую формулу и можем добавить ее в нашу таблицу производных.

Решим шестой пример и выведем еще одну формулу.

Используем правило IV и формулу 4 . Получившиеся дроби сократим.

Смотрим на данную функцию и на ее производную. Вы, конечно, поняли закономерность и готовы назвать формулу:

Учим новые формулы!

Примеры.

1. Найти приращение аргумента и приращение функции y=x 2 , если начальное значение аргумента было равно 4 , а новое –4,01 .

Решение.

Новое значение аргумента х=х 0 +Δx . Подставим данные: 4,01=4+Δх, отсюда приращение аргумента Δх =4,01-4=0,01. Приращение функции, по определению, равно разности между новым и прежним значениями функции, т.е. Δy=f (х 0 +Δх) – f (x 0). Так как у нас функция y=x 2 , то Δу =(х 0 +Δx) 2 — (х 0) 2 =(х 0) 2 +2x 0 · Δx+(Δx) 2 — (х 0) 2 =2x 0 · Δx+(Δx) 2 =

2 · 4 · 0,01+(0,01) 2 =0,08+0,0001=0,0801.

Ответ: приращение аргумента Δх =0,01; приращение функции Δу =0,0801.

Можно было приращение функции найти по-другому: Δy =y (х 0 +Δx) -y (х 0)=у(4,01) -у(4)=4,01 2 -4 2 =16,0801-16=0,0801.

2. Найти угол наклона касательной к графику функции y=f (x) в точке х 0 , если f “(х 0) = 1 .

Решение.

Значение производной в точке касания х 0 и есть значение тангенса угла наклона касательной (геометрический смысл производной). Имеем: f “(х 0) = tgα = 1 → α = 45°, так как tg45°=1.

Ответ: касательная к графику данной функции образует с положительным направлением оси Ох угол, равный 45° .

3. Вывести формулу производной функции y=x n .

Дифференцирование — это действие нахождения производной функции.

При нахождении производных применяют формулы, которые были выведены на основании определения производной, так же, как мы вывели формулу производной степени: (x n)” = nx n-1 .

Вот эти формулы.

Таблицу производных легче будет заучить, проговаривая словесные формулировки:

1. Производная постоянной величины равна нулю.

2. Икс штрих равен единице.

3. Постоянный множитель можно вынести за знак производной.

4. Производная степени равна произведению показателя этой степени на степень с тем же основанием, но показателем на единицу меньше.

5. Производная корня равна единице, деленной на два таких же корня.

6. Производная единицы, деленной на икс равна минус единице, деленной на икс в квадрате.

7. Производная синуса равна косинусу.

8. Производная косинуса равна минус синусу.

9. Производная тангенса равна единице, деленной на квадрат косинуса.

10. Производная котангенса равна минус единице, деленной на квадрат синуса.

Учим правила дифференцирования .

1. Производная алгебраической суммы равна алгебраической сумме производных слагаемых.

2. Производная произведения равна произведению производной первого множителя на второй плюс произведение первого множителя на производную второго.

3. Производная «у», деленного на «вэ» равна дроби, в числителе которой “у штрих умноженный на «вэ» минус «у, умноженный на вэ штрих», а в знаменателе — «вэ в квадрате».

4. Частный случай формулы 3.

Учим вместе!

Страница 1 из 1 1

Когда человек сделал первые самостоятельные шаги в изучении математического анализа и начинает задавать неудобные вопросы, то уже не так-то просто отделаться фразой, что «дифференциальное исчисление найдено в капусте». Поэтому настало время набраться решимости и раскрыть тайну появления на светтаблицы производных и правил дифференцирования . Начало положено в статьео смысле производной , которую я настоятельно рекомендую к изучению, поскольку там мы как раз рассмотрели понятие производной и начали щёлкать задачи по теме. Этот же урок носит ярко выраженную практическую направленность, более того,

рассматриваемые ниже примеры, в принципе, можно освоить и чисто формально (например, когда нет времени/желания вникать в суть производной). Также крайне желательно (однако опять не обязательно) уметь находить производные «обычным» методом – хотя бы на уровне двух базовых занятий: Как найти производную?и Производная сложной функции.

Но без чего-чего сейчас точно не обойтись, так это безпределов функций . Вы должны ПОНИМАТЬ, что такое предел и уметь решать их, как минимум, на среднем уровне. А всё потому, чтопроизводная

функции в точке определяется формулой:

Напоминаю обозначения и термины: называютприращением аргумента ;

– приращением функции;

– это ЕДИНЫЕ символы («дельту» нельзя «отрывать» от «икса» или «игрека»).

Очевидно, что является «динамической» переменной,– константой и результат вычисления предела– числом(иногда – «плюс» либо «минус» бесконечностью) .

В качестве точки можно рассмотреть ЛЮБОЕ значение, принадлежащееобласти определения функции, в котором существует производная.

Примечание : оговорка «в котором существует производная» –в общем случае существенна ! Так, например, точкахоть и входит в область определения функции, но производной

там не существует. Поэтому формула

не применима в точке,

и укороченная формулировка без оговорки будет некорректна. Аналогичные факты справедливы и для других функций с «обрывами» графика, в частности, для арксинуса и арккосинуса.

Таким образом, после замены , получаем вторую рабочую формулу:

Обратите внимание на коварное обстоятельство, которое может запутать чайника: в данном пределе «икс», будучи сам независимой переменной, исполняет роль статиста, а «динамику» задаёт опять же приращение . Результатом вычисления предела

является производная функция.

Исходя из вышесказанного, сформулируем условия двух типовых задач:

– Найти производную в точке , используя определение производной.

– Найти производную функцию , используя определение производной. Эта версия, по моим наблюдениям, встречается заметно чаще и ей будет уделено основное внимание.

Принципиальное отличие заданий состоит в том, что в первом случае требуется найти число (как вариант, бесконечность) , а во втором –

функцию . Кроме того, производной может и вовсе не существовать.

Как ?

Составить отношение и вычислить предел.

Откуда появилась таблица производных и правила дифференцирования? Благодаря единственному пределу

Кажется волшебством, но в

действительности – ловкость рук и никакого мошенничества. На уроке Что такое производная? я начал рассматривать конкретные примеры, где с помощью определения нашёл производные линейной и квадратичной функции. В целях познавательной разминки продолжим тревожитьтаблицу производных , оттачивая алгоритм и технические приёмы решения:

По сути, требуется доказать частный случай производной степенной функции, который обычно фигурирует в таблице: .

Решение технически оформляется двумя способами. Начнём с первого, уже знакомого подхода: лесенка начинается с дощечки, а производная функция – с производной в точке.

Рассмотрим некоторую (конкретную) точку, принадлежащуюобласти определения функции, в которой существует производная. Зададим в данной точке приращение (разумеется, не выходящее за рамки о/о -я) и составим соответствующее приращение функции:

Вычислим предел:

Неопределённость 0:0 устраняется стандартным приёмом, рассмотренным ещё в первом веке до нашей эры. Домножим

числитель и знаменатель на сопряженное выражение :

Техника решения такого предела подробно рассмотрена на вводном уроке о пределах функций .

Поскольку в качестве можно выбрать ЛЮБУЮ точкуинтервала

То, осуществив замену, получаем:

В который раз порадуемся логарифмам:

Найти производную функции , пользуясь определением производной

Решение : рассмотрим другой подход к раскрутке той же задачи. Он точно такой же, но более рационален с точки зрения оформления. Идея состоит в том, чтобы в начале решения избавиться от

подстрочного индекса и вместо буквы использовать букву.

Рассмотрим произвольную точку, принадлежащуюобласти определения функции(интервалу), и зададим в ней приращение. А вот здесь, кстати, как и в большинстве случаев, можно обойтись без всяких оговорок, поскольку логарифмическая функция дифференцируема в любой точке области определения.

Тогда соответствующее приращение функции:

Найдём производную:

Простота оформления уравновешивается путаницей, которая может

возникнуть у начинающих (да и не только). Ведь мы привыкли, что в пределе изменяется буква «икс»! Но тут всё по-другому: – античная статуя, а– живой посетитель, бодро шагающий по коридору музея. То есть «икс» – это «как бы константа».

Устранение неопределённости закомментирую пошагово:

(1) Используем свойство логарифма .

(2) В скобках почленно делим числитель на знаменатель.

(3) В знаменателе искусственно домножаем и делим на «икс» чтобы

воспользоваться замечательным пределом , при этом в качествебесконечно малой величины выступает.

Ответ : по определению производной:

Или сокращённо:

Предлагаю самостоятельно сконструировать ещё две табличные формулы:

Найти производную по определению

В данном случае составленное приращение сразу же удобно привести к общему знаменателю. Примерный образец оформления задания в конце урока (первый способ).

Найти производную по определению

А тут всё необходимо свести к замечательному пределу . Решение оформлено вторым способом.

Аналогично выводится ряд других табличных производных . Полный список можно найти в школьном учебнике, или, например, 1- м томе Фихтенгольца. Не вижу особого смысла переписывать из книг и доказательства правил дифференцирования – они тоже порождены

формулой .

Переходим к реально встречающимся заданиям: Пример 5

Найти производную функции , используя определение производной

Решение : используем первый стиль оформления. Рассмотрим некоторую точку, принадлежащую, и зададим в ней приращение аргумента. Тогда соответствующее приращение функции:

Возможно, некоторые читатели ещё не до конца поняли принцип, по которому нужно составлять приращение . Берём точку(число) и находим в ней значение функции:, то есть в функцию

вместо «икса» следует подставить. Теперь берём

Составленное приращение функции бывает выгодно сразу же упростить . Зачем? Облегчить и укоротить решение дальнейшего предела.

Используем формулы , раскрываем скобки и сокращаем всё, что можно сократить:

Индейка выпотрошена, с жаркое никаких проблем:

В итоге:

Поскольку в качестве можно выбрать любое действительное число, то проведём заменуи получим.

Ответ :по определению.

В целях проверки найдём производную с помощью правил

дифференцирования и таблицы:

Всегда полезно и приятно знать правильный ответ заранее, поэтому лучше мысленно либо на черновике продифференцировать предложенную функцию «быстрым» способом в самом начале решения.

Найти производную функции по определению производной

Это пример для самостоятельного решения. Результат лежит на поверхности:

Вернёмся к стилю №2: Пример 7

Давайте немедленно узнаем, что должно получиться. По правилу дифференцирования сложной функции :

Решение : рассмотрим произвольную точку, принадлежащую, зададим в ней приращение аргументаи составим приращение

Найдём производную:

(1) Используем тригонометрическую формулу

(2) Под синусом раскрываем скобки, под косинусом приводим подобные слагаемые.

(3) Под синусом сокращаем слагаемые, под косинусом почленно делим числитель на знаменатель.

(4) В силу нечётности синуса выносим «минус». Под косинусом

указываем, что слагаемое .

(5) В знаменателе проводим искусственное домножение, чтобы использовать первый замечательный предел . Таким образом, неопределённость устранена, причёсываем результат.

Ответ :по определению Как видите, основная трудность рассматриваемой задачи упирается в

сложность самого предела + небольшое своеобразие упаковки. На практике встречаются и тот и другой способ оформления, поэтому я максимально подробно расписываю оба подхода. Они равноценны, но всё-таки, по моему субъективному впечатлению, чайникам целесообразнее придерживаться 1-го варианта с «икс нулевым».

Пользуясь определением, найти производную функции

Это задание для самостоятельного решения. Образец оформлен в том же духе, что предыдущий пример.

Разберём более редкую версию задачи:

Найти производную функции в точке, пользуясь определением производной.

Во-первых, что должно получиться в сухом остатке? Число Вычислим ответ стандартным способом:

Решение : с точки зрения наглядности это задание значительно проще, так как в формулевместо

рассматривается конкретное значение.

Зададим в точке приращениеи составим соответствующее приращение функции:

Вычислим производную в точке:

Используем весьма редкую формулу разности тангенсов и в который раз сведём решение кпервому

замечательному пределу:

Ответ :по определению производной в точке.

Задачу не так трудно решить и «в общем виде» – достаточно заменить наили простов зависимости от способа оформления. В этом случае, понятно, получится не число, а производная функция.

Пример 10 Используя определение, найти производную функциив точке

Это пример для самостоятельного решения.

Заключительная бонус-задача предназначена, прежде всего, для студентов с углубленным изучением математического анализа, но и всем остальным тоже не помешает:

Будет ли дифференцируема функция в точке?

Решение : очевидно, что кусочно-заданная функциянепрерывна в точке, но будет ли она там дифференцируема?

Алгоритм решения, причём не только для кусочных функций, таков:

1) Находим левостороннюю производнуюв данной точке: .

2) Находим правостороннюю производнуюв данной точке: .

3) Если односторонние производныеконечны и совпадают:

, то функциядифференцируема в точкеи

геометрически здесь существует общая касательная (см. теоретическую часть урока Определение и смысл производной ).

Если получены два разных значения: (одно из которых может оказаться и бесконечным) , то функция не дифференцируема в точке.

Если же обе односторонние производные равны бесконечности

(пусть даже разных знаков), то функция не

дифференцируема в точке , но там существует бесконечная производная и общая вертикальная касательная к графику(см. Пример 5 урока Уравнение нормали ) .

Многие удивятся неожиданному расположению этой статьи в моём авторском курсе о производной функции одной переменной и её приложениях. Ведь как оно было ещё со школы: стандартный учебник в первую очередь даёт определение производной, её геометрический, механический смысл. Далее учащиеся находят производные функций по определению, и, собственно, только потом оттачивается техника дифференцирования с помощью таблицы производных .

Но с моей точки зрения, более прагматичен следующий подход: прежде всего, целесообразно ХОРОШО ПОНЯТЬ предел функции , и, в особенности, бесконечно малые величины . Дело в том, что определение производной базируется на понятии предела , которое слабо рассмотрено в школьном курсе. Именно поэтому значительная часть молодых потребителей гранита знаний плохо вникают в саму суть производной. Таким образом, если вы слабо ориентируетесь в дифференциальном исчислении либо мудрый мозг за долгие годы успешно избавился от оного багажа, пожалуйста, начните с пределов функций . Заодно освоите/вспомните их решение.

Тот же практический смысл подсказывает, что сначала выгодно научиться находить производные , в том числе производные сложных функций . Теория теорией, а дифференцировать, как говорится, хочется всегда. В этой связи лучше проработать перечисленные базовые уроки, а может и стать мастером дифференцирования , даже не осознавая сущности своих действий.

К материалам данной страницы рекомендую приступать после ознакомления со статьёй Простейшие задачи с производной , где, в частности рассмотрена задача о касательной к графику функции. Но можно и повременить. Дело в том, что многие приложения производной не требуют её понимания, и неудивительно, что теоретический урок появился достаточно поздно – когда мне потребовалось объяснять нахождение интервалов возрастания/убывания и экстремумов функции. Более того, он довольно долго находился в теме «Функции и графики », пока я всё-таки не решил поставить его раньше.

Поэтому, уважаемые чайники, не спешите поглощать суть производной, как голодные звери, ибо насыщение будет невкусным и неполным.

Понятие возрастания, убывания, максимума, минимума функции

Многие учебные пособия подводят к понятию производной с помощью каких-либо практических задач, и я тоже придумал интересный пример. Представьте, что нам предстоит путешествие в город, до которого можно добраться разными путями. Сразу откинем кривые петляющие дорожки, и будем рассматривать только прямые магистрали. Однако прямолинейные направления тоже бывают разными: до города можно добраться по ровному автобану. Или по холмистому шоссе – вверх-вниз, вверх-вниз. Другая дорога идёт только в гору, а ещё одна – всё время под уклон. Экстремалы выберут маршрут через ущелье с крутым обрывом и отвесным подъемом.

Но каковы бы ни были ваши предпочтения, желательно знать местность или, по меньшей мере, располагать её топографической картой. А если такая информация отсутствует? Ведь можно выбрать, например, ровный путь, да в результате наткнуться на горнолыжный спуск с весёлыми финнами. Не факт, что навигатор и даже спутниковый снимок дадут достоверные данные. Поэтому неплохо бы формализовать рельеф пути средствами математики.

Рассмотрим некоторую дорогу (вид сбоку):

На всякий случай напоминаю элементарный факт: путешествие происходит слева направо . Для простоты полагаем, что функция непрерывна на рассматриваемом участке.

Какие особенности у данного графика?

На интервалах функция возрастает , то есть каждое следующее её значение больше предыдущего. Грубо говоря, график идёт снизу вверх (забираемся на горку). А на интервале функция убывает – каждое следующее значение меньше предыдущего, и наш график идёт сверху вниз (спускаемся по склону).

Также обратим внимание на особые точки. В точке мы достигаем максимума , то есть существует такой участок пути, на котором значение будет самым большим (высоким). В точке же достигается минимум , и существует такая её окрестность, в которой значение самое маленькое (низкое).

Более строгую терминологию и определения рассмотрим на уроке об экстремумах функции , а пока изучим ещё одну важную особенность: на промежутках функция возрастает, но возрастает она с разной скоростью . И первое, что бросается в глаза – на интервале график взмывает вверх гораздо более круто , чем на интервале . Нельзя ли измерить крутизну дороги с помощью математического инструментария?

Скорость изменения функции

Идея состоит в следующем: возьмём некоторое значение (читается «дельта икс») , которое назовём приращением аргумента , и начнём его «примерять» к различным точкам нашего пути:

1) Посмотрим на самую левую точку: минуя расстояние , мы поднимаемся по склону на высоту (зелёная линия). Величина называется приращением функции , и в данном случае это приращение положительно (разность значений по оси – больше нуля). Составим отношение , которое и будет мерИлом крутизны нашей дороги. Очевидно, что – это вполне конкретное число, и, поскольку оба приращения положительны, то .

Внимание! Обозначение являются ЕДИНЫМ символом, то есть нельзя «отрывать» «дельту» от «икса» и рассматривать эти буквы отдельно. Разумеется, комментарий касается и символа приращения функции.

Исследуем природу полученной дроби содержательнее. Пусть изначально мы находимся на высоте 20 метров (в левой чёрной точке). Преодолев расстояние метров (левая красная линия), мы окажемся на высоте 60 метров. Тогда приращение функции составит метров (зелёная линия) и: . Таким образом, на каждом метре этого участка дороги высота увеличивается в среднем на 4 метра …не забыли альпинистское снаряжение? =) Иными словами, построенное отношение характеризует СРЕДНЮЮ СКОРОСТЬ ИЗМЕНЕНИЯ (в данном случае – роста) функции.

Примечание : числовые значения рассматриваемого примера соответствуют пропорциям чертежа лишь приблизительно.

2) Теперь пройдём то же самое расстояние от самой правой чёрной точки. Здесь подъём более пологий, поэтому приращение (малиновая линия) относительно невелико, и отношение по сравнению с предыдущим случаем будет весьма скромным. Условно говоря, метров и скорость роста функции составляет . То есть, здесь на каждый метр пути приходится в среднем пол метра подъёма.

3) Маленькое приключение на склоне горы. Посмотрим на верхнюю чёрную точку, расположенную на оси ординат. Предположим, что это отметка 50 метров. Снова преодолеваем расстояние , в результате чего оказываемся ниже – на уровне 30-ти метров. Поскольку осуществлено движение сверху вниз (в «противоход» направлению оси ), то итоговое приращение функции (высоты) будет отрицательным : метров (коричневый отрезок на чертеже). И в данном случае речь уже идёт о скорости убывания функции: , то есть за каждый метр пути этого участка высота убывает в среднем на 2 метра. Берегите одежду на пятой точке.

Теперь зададимся вопросом: какое значение «измерительного эталона» лучше всего использовать? Совершенно понятно, 10 метров – это весьма грубо. На них запросто уместится добрая дюжина кочек. Да что там кочки, внизу может быть глубокое ущелье, а через несколько метров – другая его сторона с дальнейшим отвесным подъёмом. Таким образом, при десятиметровом мы не получим вразумительной характеристики подобных участков пути посредством отношения .

Из проведённого рассуждения следует вывод – чем меньше значение , тем точнее мы опишем рельеф дороги. Более того, справедливы следующие факты:

Для любой точки подъемов можно подобрать значение (пусть и очень малое), которое умещается в границах того или иного подъёма. А это значит, что соответствующее приращение высоты будет гарантированно положительным, и неравенство корректно укажет рост функции в каждой точке этих интервалов.

– Аналогично, для любой точки склона существует значение , которое полностью уместится на этом склоне. Следовательно, соответствующее приращение высоты однозначно отрицательно, и неравенство корректно покажет убыль функции в каждой точке данного интервала.

– Особо интересен случай, когда скорость изменения функции равна нулю: . Во-первых, нулевое приращение высоты () – признак ровного пути. А во-вторых, есть другие любопытные ситуации, примеры которых вы видите на рисунке. Представьте, что судьба завела нас на самую вершину холма с парящими орлами или дно оврага с квакающими лягушками. Если сделать небольшой шажок в любую сторону, то изменение высоты будет ничтожно мало, и можно сказать, что скорость изменения функции фактически нулевая. В точках наблюдается именно такая картина.

Таким образом, мы подобрались к удивительной возможности идеально точно охарактеризовать скорость изменения функции. Ведь математический анализ позволяет устремить приращение аргумента к нулю: , то есть сделать его бесконечно малым .

По итогу возникает ещё один закономерный вопрос: можно ли для дороги и её графика найти другую функцию , которая сообщала бы нам обо всех ровных участках, подъёмах, спусках, вершинах, низинах, а также о скорости роста/убывания в каждой точке пути?

Что такое производная? Определение производной.
Геометрический смысл производной и дифференциала

Пожалуйста, прочитайте вдумчиво и не слишком быстро – материал прост и доступен каждому! Ничего страшного, если местами что-то покажется не очень понятным, к статье всегда можно вернуться позже. Скажу больше, теорию полезно проштудировать несколько раз, чтобы качественно уяснить все моменты (совет особенно актуален для студентов-«технарей», у которых высшая математика играет значительную роль в учебном процессе).

Естественно, и в самом определении производной в точке заменим на :

К чему мы пришли? А пришли мы к тому, что для функции по закону ставится в соответствие другая функция , которая называется производной функцией (или просто производной) .

Производная характеризует скорость изменения функции . Каким образом? Мысль идёт красной нитью с самого начала статьи. Рассмотрим некоторую точку области определения функции . Пусть функция дифференцируема в данной точке. Тогда:

1) Если , то функция возрастает в точке . И, очевидно, существует интервал (пусть даже очень малый), содержащий точку , на котором функция растёт, и её график идёт «снизу вверх».

2) Если , то функция убывает в точке . И существует интервал, содержащий точку , на котором функция убывает (график идёт «сверху вниз»).

3) Если , то бесконечно близко около точки функция сохраняет свою скорость постоянной. Так бывает, как отмечалось, у функции-константы и в критических точках функции , в частности в точках минимума и максимума .

Немного семантики. Что в широком смысле обозначает глагол «дифференцировать»? Дифференцировать – это значит выделить какой-либо признак. Дифференцируя функцию , мы «выделяем» скорость её изменения в виде производной функции . А что, кстати, понимается под словом «производная»? Функция произошла от функции .

Термины весьма удачно истолковывает механический смысл производной :
Рассмотрим закон изменения координаты тела , зависящий от времени , и функцию скорости движения данного тела . Функция характеризует скорость изменения координаты тела, поэтому является первой производной функции по времени: . Если бы в природе не существовало понятия «движение тела», то не существовало бы и производного понятия «скорость тела».

Ускорение тела – это скорость изменения скорости, поэтому: . Если бы в природе не существовало исходных понятий «движение тела» и «скорость движения тела», то не существовало бы и производного понятия «ускорение тела».

Вычисление производной часто встречается в заданиях ЕГЭ. Данная страница содержит список формул для нахождения производных.

Правила дифференцирования

  1. (k⋅ f(x))′=k⋅ f ′(x).
  2. (f(x)+g(x))′=f′(x)+g′(x).
  3. (f(x)⋅ g(x))′=f′(x)⋅ g(x)+f(x)⋅ g′(x).
  4. Производная сложной функции. Если y=F(u), а u=u(x), то функция y=f(x)=F(u(x)) называется сложной функцией от x. Равна y′(x)=Fu′⋅ ux′.
  5. Производная неявной функции. Функция y=f(x) называется неявной функцией, заданной соотношением F(x,y)=0, если F(x,f(x))≡0.
  6. Производная обратной функции. Если g(f(x))=x, то функция g(x) называется обратной функцией для функции y=f(x).
  7. Производная параметрически заданной функции. Пусть x и y заданы как функции от переменной t: x=x(t), y=y(t). Говорят, что y=y(x) параметрически заданная функция на промежутке x∈ (a;b), если на этом промежутке уравнение x=x(t) можно выразить в виде t=t(x) и определить функцию y=y(t(x))=y(x).
  8. Производная степенно-показательной функции. Находится путем логарифмирования по основанию натурального логарифма.
Советуем сохранить ссылку, так как эта таблица может понадобиться еще много раз.

Производная, введение и определение в 10 классе по алгебре

Дата публикации: .

Что будем изучать:
1. Введение в понятие производной.
2. Чуть-чуть истории.
3. Определение производной.
4. Производная на графике функции. Геометрический смысл производной.
5. Алгоритм нахождения производной функции.
6. Дифференцирование функции.
7. Примеры.

Введение в понятие производной


Существует множество задач совершенно разных по смыслу, но при этом есть математические модели, которые позволяют рассчитывать решения наших задач совершенно одинаковым способом. Например, если рассмотреть такие задачи как:

а) Есть некоторый счет в банке, который постоянно изменяется один раз в несколько дней, сумма постоянно растет, требуется найти с какой скоростью растет счет.
б) Завод выпускает конфеты, есть некоторый постоянный прирост выпуска конфет, найти насколько быстро увеличивается прирост конфет.
в) Скорость движения автомобиля в некоторый момент времени t, если известно положение автомобиля, и он движется по прямой линии.
г) Нам дан график функции и в некоторой точке к нему проведена касательная, требуется найти тангенс угла наклона к касательной.
Формулировка наших задач совершенно разная, и, кажется, что они решаются совершенно разными способами, но математики придумали как можно решить все эти задачи совершенно одинаковым способом. Было введено понятие производной.

Чуть-чуть истории


Термин производная ввел великий математик – Лагранж, перевод на русский язык получается из французского слова derivee, он же и ввел современные обозначения производной которые мы рассмотрим позже.
Рассматривали понятие производной в своих работах Лейбниц и Ньютон, применение нашему термину они находили в геометрии и механики соответственно.
Чуть позже мы с вами узнаем, что производная определяется через предел, но существует небольшой парадокс в истории математики. Математики научились считать производную раньше, чем ввели понятие предела и собственно поняли, что же такое производная.

Определение производной


Пусть функция y=f(x) определена на некотором интервале, содержащим внутри себя некоторую точку x0. Приращение аргумента Δx – не выходит из нашего интервала. Найдем приращение Δy и составим отношение Δy/Δx, если существует предел этого отношения при Δx стремящимся к нулю, то указанный предел называют производной функции y=f(x) в точке x0 и обозначают f’(x0).


Попробуем объяснить, что такое производная не математическим языком:
На математическом языке: производная – предел отношения приращения функции к приращению ее аргумента при стремлении приращения аргумента к нулю.
На обычном языке: производная – скорость изменения функции в точке x0.
Давайте посмотрим на графики трех функций:

Ребята, как вы думаете, какая из кривых растет быстрее?
Ответ, кажется, очевиден всем 1 кривая растет быстрее остальных. Мы смотрим, насколько круто идет вверх график функции. Другими словами — насколько быстро меняется ордината при изменении х. Одна и та же функция в разных точках может иметь разное значение производной — то есть может меняться быстрее или медленнее.

Производная на графике функции. Геометрический смысл производной


Теперь давайте посмотрим, как же найти производную с помощью графиков функции:

Посмотрим на наш график функции: Проведём в точке c абсциссой x0 касательную к графику функции. Касательная и график нашей функции соприкасаются в точке А. Нам надо оценить, насколько круто вверх идет график функции. Удобная величина для этого — тангенс угла наклона касательной.

Определение. Производная функции в точке x0 равна тангенсу угла наклона касательной, проведённой к графику функции в этой точке.

f’ (x0)=tg(α)

Угол наклона касательной выбирается как угол между касательной и положительным направлением оси абсцисс.
И так производная нашей функции равна:


И так производная в точке x0 равна тангенсу угла наклона касательной, это геометрический смысл производной.

Алгоритм нахождения производной функции


Алгоритм нахождения производной функции y=f(x).
а) Зафиксировать значение x, найти f(x).
б) Найти приращение аргумента x+ Δx, и значение приращения функции f(x+ Δx).
в) Найти приращение функции Δy= f(x+ Δx)-f(x).
г) Составить соотношение: Δy/Δx
д) Вычислить


– это и есть производная нашей функции.

Дифференцирование функции


Если функции y=f(x)имеет производную в точке x, то ее называют дифференцируемой в точке x. Процесс нахождения производной называют дифференцированием функции y=f(x).
Вернемся к вопросу непрерывности функции. Если функция дифференцируема в некоторой точке, тогда к графику функции в этой точке можно провести касательную, функция не может иметь разрыв в этой точки, тогда просто напросто нельзя провести касательную.3$.

Производная частного | Математика

При дифференцировании функций нахождение производной частного обычно вызывает наибольшие затруднения. Лучший способ разобраться и понять, как находится производная частного, — рассмотреть конкретные примеры с подробными пояснениями.

Именно этим мы сейчас и займемся. Для дифференцирования нам понадобится таблица производных. Напишем еще раз правило, по которому берется производная частного:

   

(Поначалу неплохо его выписать на листочек и держать перед глазами). В отличие от производной произведения, затруднений с определением, где здесь u,  а где — v, в производной частного нет: понятно, что все, что вверху, в числителе — это u, а все что внизу, в знаменателе — v. Если u и v — табличные функции, производная частного может быть найдена легко: достаточно расписать все по формуле, найти каждую из производных, и упростить.

Пример. Найти производную частного:

   

Здесь u=2-4x, v=3x+7

   

Производную линейной функции полезно помнить: (kx+b)’=k, где k и b — числа, причем k — число, стоящее перех x. А можно найти как производную суммы: (kx+b)’=k·x’+b’=k·1+0=k. Таким образом, (2-4x)’=-4, (3x+7)’=3, и знак умножения перед скобкой и перед буквой обычно не пишется

   

   

   

   

   

Общий множитель в числителе выносим за скобку, затем дробь сокращаем:

   

   

u=2x³+7x-5, v=6x-8. Расписываем по формуле производной частного:

   

здесь числитель представляет собой сумму и разность функций. Как находить производную суммы и разности, мы уже знаем.

   

   

   

   

Здесь u=2lnx+1, v=2√x. Значит, производная частного равна

   

   

   

   

Примеры для самопроверки. Найти производную частного:

   

Показать решение

1) u=5x²-8x, v=7-x. Теперь ищем производную частного:

   

   

   

   

   

   

   

   

Пока что мы рассмотрели только самые простые примеры на производную частного. В более сложных примерах числитель и знаменатель дроби могут быть сложными функциями, либо являться, в свою очередь, производными произведения и частного. Такие примеры мы обсудим чуть позже.

 

Производная функции. Геометрический смысл производной.

Производная функции — одна из сложных тем в школьной программе. Не каждый выпускник ответит на вопрос, что такое производная.

В этой статье просто и понятно рассказано о том, что такое производная и для чего она нужна. Мы не будем сейчас стремиться к математической строгости изложения. Самое главное — понять смысл.

Запомним определение:

Производная — это скорость изменения функции.

На рисунке — графики трех функций. Как вы думаете, какая из них быстрее растет?

Ответ очевиден — третья. У нее самая большая скорость изменения, то есть самая большая производная.

Вот другой пример.

Костя, Гриша и Матвей одновременно устроились на работу. Посмотрим, как менялся их доход в течение года:

На графике сразу все видно, не правда ли? Доход Кости за полгода вырос больше чем в два раза. И у Гриши доход тоже вырос, но совсем чуть-чуть. А доход Матвея уменьшился до нуля. Стартовые условия одинаковые, а скорость изменения функции, то есть производная, — разная. Что касается Матвея — у его дохода производная вообще отрицательна.

Интуитивно мы без труда оцениваем скорость изменения функции. Но как же это делаем?

На самом деле мы смотрим, насколько круто идет вверх (или вниз) график функции. Другими словами — насколько быстро меняется у с изменением х. Очевидно, что одна и та же функция в разных точках может иметь разное значение производной — то есть может меняться быстрее или медленнее.

Производная функции обозначается .

Покажем, как найти с помощью графика.

Нарисован график некоторой функции . Возьмем на нем точку A с абсциссой . Проведём в этой точке касательную к графику функции. Мы хотим оценить, насколько круто вверх идет график функции. Удобная величина для этого — тангенс угла наклона касательной.

Производная функции в точке  равна тангенсу угла наклона касательной, проведённой к графику функции в этой точке.

Обратите внимание — в качестве угла наклона касательной мы берем угол между касательной и положительным направлением оси .

Иногда учащиеся спрашивают, что такое касательная к графику функции. Это прямая, имеющая на данном участке единственную общую точку с графиком, причем так, как показано на нашем рисунке. Похоже на касательную к окружности.

Найдем . Мы помним, что тангенс острого угла в прямоугольном треугольнике равен отношению противолежащего катета к прилежащему. Из треугольника

Мы нашли производную с помощью графика, даже не зная формулу функции. Такие задачи часто встречаются в ЕГЭ по математике.

Есть и другое важное соотношение. Вспомним, что прямая задается уравнением

.

Величина  в этом уравнении называется угловым коэффициентом прямой. Она равна тангенсу угла наклона прямой к оси .

.

Мы получаем, что

Запомним эту формулу. Она выражает геометрический смысл производной.

Производная функции в точке  равна угловому коэффициенту касательной, проведенной к графику функции в этой точке.

Другими словами, производная равна тангенсу угла наклона касательной.

Мы уже сказали, что у одной и той же функции в разных точках может быть разная производная. Посмотрим, как же связана производная с поведением функции.

Нарисуем график некоторой функции . Пусть на одних участках эта функция возрастает, на других — убывает, причем с разной скоростью. И пусть у этой функции будут точки максимума и минимума.

В точке  функция возрастает. Касательная к графику, проведенная в точке , образует острый угол  с положительным направлением оси . Значит, в точке  производная положительна.

В точке  наша функция убывает. Касательная в этой точке образует тупой угол  с положительным направлением оси . Поскольку тангенс тупого угла отрицателен, в точке  производная отрицательна.

Вот что получается:

Если функция возрастает, ее производная положительна.

Если убывает, ее производная отрицательна.

А что же будет в точках максимума и минимума? Мы видим, что в точках  (точка максимума) и  (точка минимума) касательная горизонтальна. Следовательно, тангенс угла наклона касательной в этих точках равен нулю, и производная тоже равна нулю.

Точка  — точка максимума. В этой точке возрастание функции сменяется убыванием. Следовательно, знак производной меняется в точке  с «плюса» на «минус».

В точке  — точке минимума — производная тоже равна нулю, но ее знак меняется с «минуса» на «плюс».

Вывод: с помощью производной можно узнать о поведении функции всё, что нас интересует.

Если производная положительна, то функция возрастает.

Если производная отрицательная, то функция убывает.

В точке максимума производная равна нулю и меняет знак с «плюса» на «минус».

В точке минимума производная тоже равна нулю и меняет знак с «минуса» на «плюс».

Запишем эти выводы в виде таблицы:

возрастаетточка максимумаубываетточка минимумавозрастает
+00+

Ты нашел то, что искал? Поделись с друзьями!

Сделаем два небольших уточнения. Одно из них понадобится вам при решении задач ЕГЭ. Другое — на первом курсе, при более серьезном изучении функций и производных.

Возможен случай, когда производная функции в какой-либо точке равна нулю, но ни максимума, ни минимума у функции в этой точке нет. Это так называемая точка перегиба:

В точке  касательная к графику горизонтальна, и производная равна нулю. Однако до точки  функция возрастала — и после точки  продолжает возрастать. Знак производной не меняется — она как была положительной, так и осталась.

Бывает и так, что в точке максимума или минимума производная не существует. На графике это соответствует резкому излому, когда касательную в данной точке провести невозможно.

А как найти производную, если функция задана не графиком, а формулой? В этом случае применяется таблица производных.

Ускоренный курс по производным инструментам

Теперь позвольте мне изобразить это вместе с некоторыми моментами.

Здесь вы можете видеть, что, просто выбрав некоторую точку (равномерно расположенную) на функции, я мог найти наклон между этими точками. Однако есть несколько случаев, когда такая скорость изменения для этих точек не является хорошим представлением наклона этой функции. Да, мы можем сделать это лучше, соединив точки гораздо ближе друг к другу. Если я использую временной интервал 0,01 секунды, я получаю следующее для скорости как функции времени.

Да, это похоже на функцию косинуса, и НЕТ. Я не просто построил график функции косинуса. Фактически, это точная программа, которую я использовал для создания этой программы.

Если вы не все понимаете, не волнуйтесь. Важная часть просто проходит по точкам и вычисляет наклон («часть для i в диапазоне ..»). При желании вы можете изменить количество точек, используемых для расчета уклона – это было бы весело.

Если бы я хотел, я мог бы построить следующую функцию.

Я бы получил ТОЧНО тот же сюжет, что и выше. Итак, вы можете увидеть две вещи. Во-первых, производная – это просто скорость, с которой функция изменяется за очень крошечные промежутки времени. Во-вторых, эту производную обычно можно записать как другую фактическую математическую функцию. В общем, мы записываем производную как:

Здесь Δs заменены на d , чтобы указать, что мы смотрим на предел, когда Δt стремится к нулю. Вот и все.

А как взять производную?

Эммм… разве я только что не сделал это выше? О, вы думаете, что пользоваться компьютером – это жульничество? Хорошо, я могу это понять. Но на самом деле это не обман. Числовая программа берет производную, используя конечные (но очень крошечные) интервалы времени. В реальной жизни мы всегда имеем дело с этим, а наука имеет дело с реальным миром.

Но как получить математическую функцию без использования компьютера? Я не буду вдаваться в подробности – именно для этого и предназначен ваш урок математики. Все, что меня волнует (как тренера по физике), это то, что вы понимаете, что такое производная и как ее найти.Итак, вот несколько «правил».

Правило продукта. У вас никогда не бывает простой функции. Обычно это две перемноженные функции меньшего размера (например, a * t – даже если a является константой). Предположим, у меня есть функция g и f (обе являются функциями t ). Теперь у меня есть функция положения ( x (t) ) такая, что:

Я могу найти производную этой функции, найдя производную g (t) и f (t) в следующем манера.

Вкратце я воспользуюсь этим в примере.

Power rule. Если у вас есть многочлен, найти производную довольно просто. Предположим, у меня есть такая функция:

Где n – это просто константа. В этом случае производная этой функции будет:

Триггерные функции. Помните, я не получаю их. Я просто говорю вам «ответ» – вот производные от двух наиболее распространенных триггерных функций.

Я обманул – я пропустил небольшой шаг выше. Простите. Чтобы действительно понять триггерные производные, вам также понадобится правило цепочки.

Цепная линейка. Что делать, если у вас есть функция функции (составная функция)? Вот пример.

В этом случае я могу взять производную x как:

Это может быть немного сложнее объяснить – будем надеяться, что вы скоро охватите это в своем курсе математики.

Зачем нужны производные в физике?

Помните, что производная – это просто скорость изменения. Обычно мы думаем о скорости изменения в физике как о производной по времени. Это приводит к некоторым знакомым величинам:

Но мы используем не только производные по времени. Если вы знаете потенциальную энергию как функцию положения, вы можете найти силу, которая соответствует этому потенциалу, с пространственной производной.

Если бы вы знали функцию потенциальной энергии пружины, вы могли бы использовать ее, чтобы найти силу, прилагаемую пружиной (в направлении оси x).

На самом деле это не лучший пример, поскольку вы обычно определяете силу пружины как функцию и используете ее для получения функции потенциальной энергии пружины – но в любом случае это все еще пример.

Все эти примеры взяты из первого семестра по физике (механика и прочее). Надеюсь, к тому времени, когда вы перейдете ко второму семестру вводного курса физики, вы увидите множество производных в своем классе математики. Поверьте, есть еще много случаев, когда вам приходится использовать производные во втором семестре физики.

Последнее предупреждение. Помните, это был просто «ускоренный курс» по деривативам. Его не следует использовать вместо фактического курса математики в исчислении.

производная | Определение и факты

Производная , в математике, скорость изменения функции по отношению к переменной. Производные имеют фундаментальное значение для решения задач в области исчисления и дифференциальных уравнений. Как правило, ученые наблюдают за изменяющимися системами (динамическими системами), чтобы получить скорость изменения некоторой интересующей переменной, включить эту информацию в какое-либо дифференциальное уравнение и использовать методы интегрирования для получения функции, которую можно использовать для прогнозирования поведения исходной система в различных условиях.

Геометрически производную функции можно интерпретировать как наклон графика функции или, точнее, как наклон касательной в точке. Фактически, его расчет происходит из формулы наклона прямой линии, за исключением того, что для кривых необходимо использовать процесс ограничения. Наклон часто выражается как «подъем» по сравнению с «пробегом» или, в декартовых терминах, отношение изменения y к изменению x . Для прямой, показанной на рисунке, формула наклона имеет вид ( y 1 y 0 ) / ( x 1 x 0 ).Другой способ выразить эту формулу: [ f ( x 0 + h ) – f ( x 0 )] / h , если h используется для x 1 x 0 и f ( x ) для y . Это изменение обозначений полезно для перехода от идеи наклона прямой к более общей концепции производной функции.

Британская викторина

Определить: математические термины

Вот ваша миссия, если вы решите принять ее: Определите следующие математические термины до того, как истечет время.

Для кривой это соотношение зависит от того, где выбраны точки, что отражает тот факт, что кривые не имеют постоянного наклона. Чтобы найти наклон в желаемой точке, выбор второй точки, необходимой для расчета отношения, представляет собой трудность, потому что, как правило, отношение будет представлять только средний наклон между точками, а не фактический наклон в любой точке ( см. рисунок ). Чтобы обойти эту трудность, используется процесс ограничения, при котором вторая точка не фиксируется, а задается переменной, например h в соотношении для прямой линии выше.Нахождение предела в этом случае – это процесс нахождения числа, к которому отношение приближается, когда h приближается к 0, так что предельное отношение будет представлять фактический наклон в данной точке. Некоторые манипуляции нужно проделать с частным [ f ( x 0 + h ) – f ( x 0 )] / h , чтобы его можно было переписать в виде в котором предел h приближается к 0, можно увидеть более прямо. Рассмотрим, например, параболу: x 2 .При нахождении производной x 2 , когда x равно 2, частное будет [(2 + h ) 2 – 2 2 ] / h . При расширении числителя частное становится (4 + 4 h + h 2 -4) / h = (4 h + h 2 ) / h . И числитель, и знаменатель по-прежнему приближаются к 0, но если h на самом деле не равно нулю, а очень близко к нему, тогда h можно разделить, давая 4 + h , что легко увидеть, что оно приближается к 4 как h. приближается к 0.

наклон кривой

Наклон или мгновенная скорость изменения кривой в определенной точке ( x 0 , f ( x 0 )) можно определить, соблюдая предел средней скорости изменения, когда вторая точка ( x 0 + h , f ( x 0 + h )) приближается к исходной точке.

Encyclopædia Britannica, Inc.

Подводя итог, производная f ( x ) при x 0 , записывается как f ′ ( x 0 ), ( d f / d x ) ( x 0 ) или D f ( x 0 ), определяется, как если бы этот предел существует.

Получите подписку Britannica Premium и получите доступ к эксклюзивному контенту. Подпишитесь сейчас

Дифференциация, то есть вычисление производной, редко требует использования базового определения, но вместо этого может быть достигнута посредством знания трех основных производных, использования четырех правил работы и знания того, как манипулировать функциями.

Развитие интуиции для производных – лучше объяснение

Как вы хотите, чтобы вам объяснили производную? Вот мое мнение.

Psst! Производная – это сердце исчисления, похороненное внутри этого определения:

Но что это значит?

Допустим, я дал вам волшебную газету, в которой перечислялись ежедневные изменения фондового рынка на следующие несколько лет (+ 1% в понедельник, -2% во вторник …). Что ты мог сделать?

Что ж, вы бы применили изменения одно за другим, наметили будущие цены и купили дешево / продали дорого, чтобы построить свою империю. Вы даже можете перестать использовать обезьян, которые выбирают случайные акции для вашего портфеля.

Подобно этой волшебной газете, производным является хрустальный шар, который точно объясняет, как изменится узор. Зная это, вы можете строить график прошлого / настоящего / будущего, находить минимумы / максимумы и, следовательно, принимать более правильные решения. Это довольно интересно, больше, чем типичное описание «производная – это наклон функции».

Давайте отойдем от корявого уравнения. Уравнения существуют для передачи идей: поймите идею, а не грамматику.

Производные финансовые инструменты создают идеальную модель изменений на основе неточного предположения.

Этот результат явился результатом тысяч лет размышлений от Архимеда до Ньютона. Давайте посмотрим на аналогии, стоящие за этим.

Мы все живем в сияющем континууме

Бесконечность – постоянный источник парадоксов («головной боли»):

  • Линия состоит из точек? Конечно.
  • Значит, на прямой бесконечное количество точек? Ага.
  • Как пройти через комнату, если нужно посетить бесконечное количество точек? (Гы, спасибо Зено).

И все же мы переезжаем. Моя интуиция – бороться с бесконечностью с бесконечностью. Конечно, между 0 и 1 есть бесконечность точек. Но я перемещаю на две бесконечности на точек в секунду (каким-то образом!) И пересекаю промежуток за полсекунды.

Расстояние имеет бесконечное количество точек, движение возможно, поэтому движение выражается в «бесконечности точек в секунду».

Вместо того, чтобы думать о различиях («Как далеко до следующей точки?»), Мы можем сравнивать скорости («Как быстро вы продвигаетесь через этот континуум?»).

Это странно, но вы можете увидеть 10/5 как «Мне нужно пройти 10« бесконечностей »за 5 сегментов времени. Для этого я путешествую на 2« бесконечности »за каждую единицу времени».

Аналогия: смотрите разделение как скорость движения через континуум точек

Что после нуля?

Еще одна головоломка: какое число стоит после нуля? .01? .0001?

грн. Все, что вы можете назвать, я могу назвать меньшим (я просто уменьшу ваше число вдвое … нет!).

Хотя мы не можем вычислить число после нуля, оно должно быть там, верно? Подобно демонам прошлого, это «число, которое нельзя записать, чтобы не быть пораженным».

Назовите пробел до следующего числа $ dx $. Я не знаю точно, насколько он большой, но он есть!

Аналогия: dx – это «прыжок» к следующему числу в континууме.

Размеры зависят от прибора

Производная предсказывает изменение. Хорошо, как мы измеряем скорость (изменение расстояния)?

Офицер: Вы знаете, как быстро вы двигались?

Водитель: Понятия не имею.

Офицер: 95 миль в час.

Водитель: Но я уже час не езжу!

Нам явно не нужен «полный час», чтобы измерить вашу скорость.Мы можем провести измерения до и после (скажем, более 1 секунды) и получить мгновенную скорость. Если вы переместитесь на 140 футов за одну секунду, вы разгонитесь до ~ 95 миль в час. Все просто, правда?

Не совсем так. Представьте себе видеокамеру, направленную на Кларка Кента (альтер-эго Супермена). Камера записывает 24 кадра в секунду (40 мс на фотографию), и Кларк кажется неподвижным. Посекундно он не движется и его скорость составляет 0 миль в час.

Снова неверно! Между каждой фотографией, в течение этих 40 мс, Кларк превращается в Супермена, раскрывает преступления и возвращается к своему креслу, чтобы сделать красивую фотографию.Мы измерили 0 миль в час, но он действительно движется – он движется слишком быстро для наших инструментов!

Аналогия: Как камера, наблюдающая за Суперменом, скорость, которую мы измеряем, зависит от инструмента!

Беговая дорожка

Мы приближаемся к жевательному, слегка острому центру производной. Чтобы обнаружить изменения, нам нужны измерения до и после, но наши измерения могут быть ошибочными.

Представьте Санту без рубашки на беговой дорожке (давай, я подожду). Мы собираемся измерить его пульс в ходе стресс-теста: мы прикрепляем десятки тяжелых холодных электродов и заставляем его бегать трусцой.

Санта фыркает, он пыхтит, и его пульс достигает 190 ударов в минуту. Это должно быть его пульс “под стрессом”, верно?

Нет. Видите, само присутствие суровых ученых и холодных электродов увеличивало его сердцебиение! Мы измерили 190 ударов в минуту, но кто знает, что мы бы увидели, если бы электродов не было! Конечно, если бы электродов не было, у нас не было бы измерения.

Что делать? Ну посмотрите на систему:

  • измерение = фактическое количество + эффект измерения

Ач.После множества исследований мы можем обнаружить: «О, каждый электрод увеличивает частоту сердечных сокращений на 10 ударов в минуту». Делаем замер (неполное предположение 190) и убираем влияние электродов («точная оценка»).

Аналогия: Удалите «электродный эффект» после проведения измерения

Кстати, “электродный эффект” проявляется везде. Исследования показывают эффект Хоторна, когда люди меняют свое поведение , потому что их изучают . Ну и дела, кажется, все, кого мы изучаем, придерживаются своей диеты!

Понимание производного инструмента

Вооружившись этими идеями, мы можем увидеть, как меняются производные модели:

Начните с какой-нибудь системы для изучения, $ f (x) $:

  1. Изменение на минимально возможную сумму (dx $)
  2. Получите разницу до и после: $ f (x + dx) – f (x) $
  3. Мы не знаем точно, насколько мала $ dx $, и нам все равно: получаем скорость движения через континуум: $ [f (x + dx) – f (x)] / dx $
  4. Эта скорость, даже небольшая, имеет некоторую ошибку (наши камеры слишком медленные!).Предскажите, что произойдет, если измерения будут идеальными, если $ dx $ не будет.

Магия на последнем этапе: как удалить электроды? У нас есть два подхода:

  • Пределы: что происходит, когда $ dx $ превращается в ничто, за пределами погрешности?
  • Бесконечно малые: что, если $ dx $ – крошечное число, не обнаруживаемое в нашей системе счисления?

Оба способа формализовать понятие «Как нам выбросить $ dx $, когда он не нужен?».

Моя любимая мозоль: пределы – это современный формализм, их не существовало во времена Ньютона.2 = 5 $). Из чего сделана цифра «5»?

  • Измеренная скорость = Фактическая скорость + Ошибка
  • $ 5 = 2x + dx $
  • 5 долларов = 2 (2) + 1 90 349 долларов

Конечно, мы измерили «5 единиц, перемещаемых в секунду», потому что мы перешли от 4 до 9 за один интервал. Но наши инструменты обманывают нас! 4 единицы скорости возникли из-за реального изменения и 1 единица из-за некачественных инструментов (1,0 – это большой скачок, не так ли?).

Если мы ограничимся целыми числами, 5 будет идеальным измерением скорости от 4 до 9. Нет “ошибки” в предположении $ dx = 1 $, потому что это истинный интервал между соседними точками.

Но в реальном мире измерения каждые 1,0 секунды – это слишком медленно. Что, если бы наш $ dx $ был 0,1? Какую скорость мы бы измерили при $ x = 2 $?

Итак, мы исследуем изменение с $ x = 2 $ на $ x = 2,1 $:

Помните, 0,41 – это то, что мы изменили в интервале 0,1. Наша скорость на единицу равна 0,41 / 0,1 = 4,1. И снова имеем:

  • Измеренная скорость = Фактическая скорость + Ошибка
  • $ 4,1 = 2x + dx $

Интересно. При $ dx = 0,1 $ измеренная и фактическая ставки близки (4.2 $ изменений, нашел “несовершенное” измерение $ 2x + dx $ и вывел “идеальную” модель изменения как $ 2x $.

Производная как «непрерывное деление»

Я считаю интеграл лучшим умножением, при котором вы можете применить изменяющуюся величину к другой.

Производная – «лучшее деление», когда вы получаете скорость через континуум в каждый момент. Пример 10/5 = 2 говорит о том, что “у вас постоянная скорость 2 в континууме”.

Когда ваша скорость меняется по ходу движения, вам нужно описывать свою скорость в каждый момент.2 $, при $ x = 22 $ мы меняемся на 44 (конкретная скорость изменения).

  • «Производная $ dx $» может относиться к крошечному гипотетическому переходу к следующей позиции. Технически $ dx $ – это «дифференциал», но термины путаются. Иногда люди говорят «производная от $ x $» и имеют в виду $ dx $.

  • Попался: наши модели могут быть не идеальными

    Мы нашли «идеальную» модель, произведя измерения и улучшив ее. Иногда этого недостаточно – мы прогнозируем, что произойдет с , если $ dx $ не было, но добавили $ dx $, чтобы получить наше первоначальное предположение!

    Некоторые некорректные функции не поддаются предсказанию: есть разница между удалением $ dx $ с ограничением и тем, что на самом деле происходит в этот момент.Они называются «прерывистыми» функциями, которые, по сути, «не могут быть смоделированы с ограничениями». Как вы можете догадаться, производная от них не работает, потому что мы не можем предсказать их поведение.

    Прерывистые функции на практике встречаются редко и часто существуют как “Попался!” контрольные вопросы («О, вы пытались взять производную от разрывной функции, вы потерпели неудачу»). Осознайте теоретические ограничения производных, а затем осознайте их практическое использование для измерения любых природных явлений.Почти каждая функция, которую вы увидите (синус, косинус, е, многочлены и т. Д.), Является непрерывной.

    Попался: интеграции на самом деле не существует

    Взаимосвязь между производными, интегралами и антипроизводными имеет нюансы (и изначально я ошибся). Вот метафора. Начните с тарелки, ваша функция для изучения:

    • Дифференциация разбивает тарелку на осколки. Существует определенная процедура: возьмите разницу, найдите скорость изменения, а затем предположите, что $ dx $ нет.
    • Интеграция взвешивает осколки: ваша исходная функция была «такой» большой. 2 $ не одно и то же? Да, но это не очевидно: это теорема Пифагора!»).Спасибо Джошуа Цукеру за то, что помог разобраться со мной.

      Чтение по математике

      Математика – это язык, и я хочу «читать» исчисление (а не «декламировать» исчисление, то есть как мы можем декламировать средневековые немецкие гимны). Мне нужен смысл определений.

      Моя самая большая ага! осознавал временную роль $ dx $: он производит измерение и удаляется, чтобы сделать идеальную модель. Пределы / бесконечно малые – это формализм, мы не можем зацикливаться на них. Казалось, Ньютон без них обходился нормально.

      Вооружившись этими аналогиями, другие математические вопросы станут интересными:

      • Как измерить бесконечность различных размеров? (В некотором смысле они все «бесконечны», в других смыслах диапазон (0,1) меньше, чем (0,2))
      • Каковы настоящие правила того, как заставить $ dx $ «уйти»? (Как на самом деле работают бесконечно малые и пределы?)
      • Как описывать числа, не записывая их? «Следующее число после 0» – это начало анализа (которому я хочу научиться).

      Основы интересны, когда вы понимаете, почему они существуют. Счастливая математика.

      Другие сообщения из этой серии

      1. Нежное введение в изучение исчисления
      2. Понимание исчислений с помощью метафоры банковского счета
      3. Доисторическое исчисление: открытие числа Пи
      4. Аналогия с исчислением: интегралы как умножение
      5. Исчисление: построение интуиции для производных
      6. Как понять деривативы: правила продукта, власти и цепочки
      7. Как понимать производные: правило частного, экспоненты и логарифмы
      8. Интуитивное введение в ограничения
      9. Интуиция к серии Тейлора (аналогия с ДНК)
      10. Зачем нужны пределы и бесконечно малые?
      11. Обучение исчислению: преодоление нашей искусственной потребности в точности
      12. Дружеский чат о том, 0.999 … = 1
      13. Аналогия: камера исчисления
      14. Практика абстракции: графы исчисления
      15. Quick Insight: более простая арифметика с исчислением
      16. Как сложить от 1 до 100 с помощью исчисления
      17. Интеграл греха (x): геометрическая интуиция

      Производная (математика) – Простая английская Википедия, бесплатная энциклопедия

      Функция (черный) и касательная (красный). Производная в точке – это наклон касательной.

      В математике (особенно в дифференциальном исчислении) производная – это способ показать мгновенную скорость изменения: то есть величину, на которую функция изменяется в одной заданной точке.Для функций, которые действуют на действительные числа, это наклон касательной в точке на графике. Производная часто записывается как dydx {\ displaystyle {\ tfrac {dy} {dx}}} («dy over dx», что означает разность по y, разделенная на разность по x). d не является переменной и поэтому не может быть отменен. Другое распространенное обозначение – это f ′ (x) {\ displaystyle f ‘(x)} – производная функции f {\ displaystyle f} в точке x {\ displaystyle x}. [1] [2] [3]

      Анимация, дающая интуитивное представление о производной, поскольку “качели” функции меняются при изменении аргумента.

      Производная y по x определяется как изменение y по сравнению с изменением x, поскольку расстояние между x0 {\ displaystyle x_ {0}} и x1 {\ displaystyle x_ {1}} становится бесконечно малым (бесконечно малым ). С математической точки зрения, [2] [3]

      f ‘(a) = limh → 0f (a + h) −f (a) h {\ displaystyle f’ (a) = \ lim _ {h \ to 0} {\ frac {f (a + h) -f (a)} {h}}}

      То есть, когда расстояние между двумя точками x (h) становится ближе к нулю, наклон линии между ними приближается к касательной.

      Линейные функции [изменить | изменить источник]

      Производные линейных функций (функции вида mx + c {\ displaystyle mx + c} без квадратичных или более высоких членов) постоянны. То есть производная в одном месте графика останется прежней в другом.

      Когда зависимая переменная y {\ displaystyle y} напрямую принимает значение x {\ displaystyle x} (y = x {\ displaystyle y = x}), наклон линии равен 1 во всех местах, поэтому ddx ( x) = 1 {\ displaystyle {\ tfrac {d} {dx}} (x) = 1} независимо от того, где находится позиция.{2}}}

      Логарифмические функции [изменить | изменить источник]

      Производная логарифмов является обратной величиной: [2]

      ddxln⁡ (x) = 1x {\ displaystyle {\ frac {d} {dx}} \ ln (x) = {\ frac {1} {x}}}.

      Возьмем, например, ddxln⁡ (5x) {\ displaystyle {\ frac {d} {dx}} \ ln \ left ({\ frac {5} {x}} \ right)}. Это можно свести к (по свойствам логарифмов):

      ddx (ln⁡ (5)) – ddx (ln⁡ (x)) {\ displaystyle {\ frac {d} {dx}} (\ ln (5)) – {\ frac {d} {dx}} (\ ln (x))}

      Логарифм 5 является константой, поэтому его производная равна 0.Производная от ln⁡ (x) {\ displaystyle \ ln (x)} равна 1x {\ displaystyle {\ tfrac {1} {x}}}. Так,

      0 − ddxln⁡ (x) = – 1x {\ displaystyle 0 – {\ frac {d} {dx}} \ ln (x) = – {\ frac {1} {x}}}

      Для производных логарифмов не по основанию e , например ddx (log10⁡ (x)) {\ displaystyle {\ tfrac {d} {dx}} (\ log _ {10} (x))}, это можно свести к :

      ddxlog10⁡ (x) = ddxln⁡xln⁡10 = 1ln⁡10ddxln⁡x = 1xln⁡ (10) {\ displaystyle {\ frac {d} {dx}} \ log _ {10} (x) = {\ frac {d} {dx}} {\ frac {\ ln {x}} {\ ln {10}}} = {\ frac {1} {\ ln {10}}} {\ frac {d} {dx} } \ ln {x} = {\ frac {1} {x \ ln (10)}}}

      Тригонометрические функции [изменить | изменить источник]

      Функция косинуса является производной функции синуса, а производная косинуса – отрицательным синусом (при условии, что x измеряется в радианах): [2]

      ddxsin⁡ (x) = cos⁡ (x) {\ displaystyle {\ frac {d} {dx}} \ sin (x) = \ cos (x)}
      ddxcos⁡ (x) = – sin⁡ (x) {\ displaystyle {\ frac {d} {dx}} \ cos (x) = – \ sin (x)}
      ddxsec⁡ (x) = sec⁡ (x) tan⁡ (x) {\ displaystyle {\ frac {d} {dx}} \ sec (x) = \ sec (x) \ tan (x)}.{5} + 2x \,}

      Производная функции может использоваться для поиска максимумов и минимумов функции путем поиска мест, где ее наклон равен нулю.

      Производные используются в методе Ньютона, который помогает найти нули (корни) функции. Можно также использовать производные для определения вогнутости функции и определения того, увеличивается или уменьшается функция.

      1. «Список математических и аналитических символов». Математическое хранилище .2020-05-11. Проверено 15 сентября 2020.
      2. 2,0 2,1 2,2 2,3 2,4 Weisstein, Eric W. “Derivative”. mathworld.wolfram.com . Проверено 15 сентября 2020.
      3. 3,0 3,1 «Значение производной – подход к исчислению». themathpage.com . Проверено 15 сентября 2020.

      Деривативы для чайников

      Я упомянул деривативы, но сначала позвольте мне ответить на вопрос попроще.


      Короткие продажи

      Здесь, наверное, все понимают, как работает фондовый рынок. Вы можете покупать или продавать акции. В зависимости от того, как рынок относится к акциям, он идет вверх или вниз. Основная идея состоит в том, чтобы купить акцию, когда цена низкая, и продать ее, когда она станет высокой. В этом базовом понимании не так уж много места для ставок ни против чего. Здесь нас с вами выгнали из комнаты, и началась настоящая сделка.

      Если вы большой любитель, для вас будут особые правила.Смотрите, брокеры – это ребята, которые покупают и продают акции. В любое время они держат тонны запасов для своих клиентов. Итак, допустим, вы – большая шишка, и вы дружите с кучей брокеров. Вы смотрите на акции, которые они держат, и думаете, что они скоро потеряют свою ценность.

      Вы можете одолжить у них акции за определенную плату. Брокер позволяет вам занять его, потому что они не зарабатывают на этом деньги, просто сидя там. Теперь вы назначаете срок, на который вы занимаетесь.

      После того, как вы одалживаете акции, вы продаете их. Теперь у вас есть деньги. Затем вы ждете, пока акции упадут в цене. Когда цена упадет настолько низко, насколько вы думаете, вы снова покупаете акции. Теперь вы возвращаете акции брокеру. Вы сохраняете разницу между ценой, по которой вы ее продали, и ценой, по которой вы ее выкупили.

      Теперь нужно понять несколько вещей:

      -Если вы следуете традиционной модели покупки по низкой цене / продажи по высокой, максимум, что вы можете потерять, – это стоимость акций.

      – Продавцы коротких позиций берут на себя бесконечный риск, потому что, если акции растут, они теряют разницу.

      – Брокеры также берут на себя риск в этом уравнении, потому что на самом деле они не владеют акциями, которые они ссужают. Если эта акция растет и владелец хочет продать, ему теперь нужно в спешке вернуть акции.

      Из-за этих проблем существуют некоторые меры защиты от коротких продаж. На большинстве фондовых рынков любые предоставленные в кредит акции публично котируются вместе с акциями. Это означает, что покупатели знают, когда кто-то пытается продать акции.Также существуют правила для брокера относительно выдачи чужих акций в долг.

      Плохая часть коротких продаж состоит в том, что никто не может этого сделать. Как я уже сказал, это только клуб хайроллеров. Часто заемные акции поступают из пенсионных фондов, где есть много акций, которые можно сдать в аренду, а владелец не следит за акциями.

      Хорошо, вот как вы будете делать ставки против акции. Допустим, вы хотите сделать ставку на что-то еще, например, на цыплят, ипотеку или Грецию. Для этого нам нужно войти в тени.


      Деривативы

      Производный инструмент – это не акция или облигация. Это не баррель масла и не бушель кукурузы. Нет рынка, где можно купить или продать деривативы. Добро пожаловать в самое большое и в то же время самое уютное казино в мире.

      Проще говоря, производная – это контракт между двумя людьми. Контракт может быть неправильным словом. Думайте об этом больше как о пари между двумя людьми, которое подкреплено судебными исками. Производная не имеет собственной стоимости.Вместо этого он основан на ценности чего-то другого. Что-нибудь еще.

      Допустим, вы выращиваете брокколи. Когда ваш урожай поступит, вы отнесете его дистрибьютору брокколи, который заплатит вам текущую рыночную цену за вашу брокколи. В некоторые годы вы получаете действительно хорошую цену, а в некоторые – плохую. Через несколько лет вы чувствуете себя комфортно, гадая, какой будет цена при сборе урожая.

      Значит, вы заключаете пари с дистрибьютором за несколько месяцев вперед. Вы убеждаете их купить брокколи по 10 долларов за бушель.Они соглашаются с такой ценой. Вы только что создали производную. Такая производная называется будущим. Если пришел урожай, то текущая цена составляет 8 долларов за бушель, и вы сделаете эту ставку впереди. Если рыночная цена составляет 15 долларов за бушель, дистрибьютор выигрывает. Вы оба по контракту обязаны соблюдать соглашение.

      Фьючерсы – это только один из видов производных финансовых инструментов. Фьючерсы также имеют решающее значение для многих фермеров, продолжающих вести бизнес. Не все производные – это плохо. Для фермера, правильно составляющего годовой бюджет, гарантированная цена стоит риска.

      Однако некоторые производные могут быть очень плохими. Например, AIG предлагала производные финансовые инструменты на основе стоимости ценных бумаг, обеспеченных ипотекой. Они согласились заплатить другим людям, если эти ценные бумаги потеряют ценность. Если они приобретали ценность, люди должны были платить им. Этот вид производного инструмента называется свопом или свопом по кредиту.

      Это очень странная особенность деривативов. Ни одна из двух сторон не должна фактически владеть предметом, на который они делают ставки. Я действительно мог бы сделать производную от брокколи, не будучи фермером или дистрибьютором.Нет никаких правил о том, как должна выглядеть производная. Мне также не нужно каким-либо образом сообщать о существовании производного инструмента.

      Итак, когда дело доходит до рынков облигаций в Греции, я могу делать ставки против них, используя дериватив. Вместо коротких продаж, как я показал выше, мне просто нужно создать или купить производный инструмент на основе стоимости этой облигации. Если я думаю, что Греция терпит крах, я могу купить своп на стоимость облигации. Как я уже сказал, на самом деле мне не нужно владеть облигацией.Я просто нахожу того, кто готов сделать ставку в противоположном мне направлении.

      Спекуляция

      Этот теперь абсурдно длинный и, без сомнения, игнорируемый дневник вращается вокруг концепции спекуляций. Спекуляция – это часть ведения бизнеса, но со спекуляцией есть проблема. Поскольку так много спекуляций делается в тени, мы наблюдаем усиление нисходящих колебаний в экономике. У Греции действительно были серьезные проблемы с государственным дефицитом, но спекуляции, скорее всего, ускорили ее упадок.

      Проблема в том, что все эти предположения происходят без ведома общественности. Именно здесь казино могут начать возмущаться сравнением с Уолл-стрит. По крайней мере, в казино все имеют одинаковые возможности делать ставки, и все фишки лежат на столе у ​​всех на виду.

      Определение производной

      Что такое производная?

      Производный инструмент – это финансовая ценная бумага, стоимость которой зависит от базового актива или группы активов или получена из них – эталон.Сам производный инструмент представляет собой договор между двумя или более сторонами, и цена производного инструмента определяется колебаниями базового актива.

      Наиболее распространенными базовыми активами для деривативов являются акции, облигации, сырьевые товары, валюты, процентные ставки и рыночные индексы. Эти активы обычно покупаются через брокерских контор.

      (Посмотрите, как ваш брокер сравнивается со списком лучших онлайн-брокеров Investopedia).

      Мелисса Линг {Copyright} Investopedia, 2019.

      Деривативы могут торговаться вне биржи (OTC) или на бирже. Внебиржевые деривативы составляют большую долю на рынке деривативов. Внебиржевые деривативы, как правило, имеют большую вероятность контрагентского риска. Риск контрагента – это опасность неисполнения обязательств одной из сторон, участвующих в сделке. Эти стороны торгуют между двумя частными сторонами и не регулируются.

      И наоборот, производные финансовые инструменты, которые торгуются на бирже, стандартизированы и более жестко регулируются.

      Производный инструмент: My Favorite Financial Term

      Основы производных инструментов

      Деривативы могут использоваться для хеджирования позиции, спекуляции на направленном движении базового актива или увеличения кредитного плеча для холдингов. Их стоимость зависит от колебаний стоимости базового актива.

      Первоначально деривативы использовались для обеспечения сбалансированных обменных курсов для товаров, продаваемых на международном уровне. Из-за разницы в стоимости национальных валют международным трейдерам требовалась система для учета различий.Сегодня деривативы основаны на широком спектре транзакций и имеют гораздо больше применений. Существуют даже производные, основанные на погодных данных, таких как количество дождя или количество солнечных дней в регионе.

      Например, представьте европейского инвестора, все инвестиционные счета которого номинированы в евро (EUR). Этот инвестор покупает акции американской компании через биржу США за доллары США (USD). Теперь инвестор подвергается валютному риску, пока владеет этими акциями.Риск обменного курса – это угроза увеличения стоимости евро по отношению к доллару США. Если стоимость евро растет, любая прибыль, которую инвестор получает при продаже акций, становится менее ценной, когда они конвертируются в евро.

      Чтобы застраховаться от этого риска, инвестор мог купить производную валюту, чтобы зафиксировать определенный обменный курс. Производные финансовые инструменты, которые можно использовать для хеджирования такого рода рисков, включают валютные фьючерсы и валютные свопы.

      Спекулянт, который ожидает повышения курса евро по сравнению с долларом, может получить прибыль, используя производный инструмент, стоимость которого растет вместе с евро.При использовании деривативов для спекуляции на движении цены базового актива инвестору не обязательно иметь холдинг или присутствие в портфеле базового актива.

      Ключевые выводы

      • Производные инструменты – это ценные бумаги, стоимость которых определяется базовым активом или эталоном.
      • Общие производные инструменты включают фьючерсные контракты, форварды, опционы и свопы.
      • Большинство деривативов не торгуются на биржах и используются организациями для хеджирования рисков или спекуляции на изменениях цен на базовый актив.
      • Биржевые деривативы, такие как фьючерсы или опционы на акции, стандартизированы и устраняют или снижают многие риски внебиржевых деривативов
      • Деривативы обычно представляют собой инструменты с кредитным плечом, что увеличивает их потенциальные риски и выгоды.

      Общие формы производных инструментов

      Существует множество различных типов производных финансовых инструментов, которые можно использовать для управления рисками, для спекуляций и увеличения позиций. Деривативы – это растущий рынок, предлагающий продукты, которые удовлетворяют практически любые потребности или допускают риск.

      Фьючерс

      Фьючерсные контракты, также известные как фьючерсы, представляют собой соглашение между двумя сторонами о покупке и поставке актива по согласованной цене в будущем. Торговля фьючерсами на бирже, контракты стандартизированы. Трейдеры будут использовать фьючерсные контракты, чтобы хеджировать свой риск или спекулировать ценой базового актива. Стороны, участвующие в фьючерсной сделке, обязаны выполнить обязательство по покупке или продаже базового актива.

      Например, предположим, что 6 ноября 2019 года Компания-А покупает фьючерсный контракт на нефть по цене 62,22 доллара за баррель, который истекает 19 декабря 2019 года. Компания делает это, потому что ей нужна нефть в декабре, и она обеспокоена тем, что цена вырастет до того, как компании потребуется покупка. Покупка нефтяного фьючерсного контракта хеджирует риск компании, потому что продавец на другой стороне контракта обязан поставить нефть Компании А по цене 62,22 доллара за баррель после истечения срока контракта. Предположим, что к декабрю цены на нефть вырастут до 80 долларов за баррель.19, 2019. Компания-А может принять поставку нефти от продавца фьючерсного контракта, но если она больше не нуждается в нефти, она также может продать контракт до истечения срока и сохранить прибыль.

      В этом примере возможно, что и покупатель фьючерса, и продавец хеджировали риск. Компания-А нуждалась в нефти в будущем и хотела компенсировать риск того, что цена может вырасти в декабре, с помощью длинной позиции по фьючерсному контракту на нефть. Продавцом может быть нефтяная компания, которая была обеспокоена падением цен на нефть и хотела устранить этот риск путем продажи или «короткой продажи» фьючерсного контракта, фиксировавшего цену, которую она получит в декабре.

      Также возможно, что продавец или покупатель – или оба – участников нефтяных фьючерсов были спекулянтами с противоположным мнением относительно направления декабрьской нефти. Если бы стороны, участвующие в фьючерсном контракте, были спекулянтами, маловероятно, что кто-либо из них захотел бы договориться о поставке нескольких баррелей сырой нефти. Спекулянты могут прекратить свое обязательство по покупке или доставке базового товара, закрыв – раскрутив – свой контракт до истечения срока его действия с помощью компенсирующего контракта.

      Например, фьючерсный контракт на нефть West Texas Intermediate (WTI) на CME представляет собой 1000 баррелей нефти. Если цена на нефть вырастет с 62,22 доллара до 80 долларов за баррель, трейдер с длинной позицией – покупатель – по фьючерсному контракту получил бы прибыль 17 780 долларов [(80 долларов – 62,22 доллара) X 1000 = 17 780 долларов]. У трейдера с короткой позицией – продавца – в контракте убыток составит 17 780 долларов.

      Не все фьючерсные контракты рассчитываются по истечении срока путем поставки базового актива.Расчет по многим производным финансовым инструментам осуществляется наличными, что означает, что прибыль или убыток от сделки – это просто учетный денежный поток на брокерском счете трейдера. Фьючерсные контракты с расчетами наличными включают многие фьючерсы на процентную ставку, фьючерсы на фондовые индексы и более необычные инструменты, такие как фьючерсы на волатильность или погодные фьючерсы.

      Нападающие

      Форвардные контракты, известные просто как форвардные, похожи на фьючерсы, но торгуются не на бирже, а только на внебиржевом рынке. При создании форвардного контракта покупатель и продавец могут настроить условия, размер и процесс расчета по производному инструменту.Будучи внебиржевыми продуктами, форвардные контракты несут более высокий риск контрагента как для покупателей, так и для продавцов.

      Риски контрагента – это своего рода кредитный риск, заключающийся в том, что покупатель или продавец могут быть не в состоянии выполнить обязательства, изложенные в контракте. Если одна сторона контракта становится неплатежеспособной, другая сторона может не иметь права регресса и может потерять ценность своего положения. После создания стороны форвардного контракта могут компенсировать свою позицию с другими контрагентами, что может увеличить потенциальные риски контрагента по мере того, как в один и тот же контракт вовлекается больше трейдеров.

      Свопы

      Свопы – еще один распространенный тип производных финансовых инструментов, часто используемых для обмена одного вида денежных потоков на другой. Например, трейдер может использовать процентный своп, чтобы переключиться с ссуды с переменной процентной ставкой на ссуду с фиксированной процентной ставкой или наоборот.

      Представьте, что компания XYZ взяла заем в размере 1 000 000 долларов США и выплачивает переменную процентную ставку по ссуде, которая в настоящее время составляет 6%. XYZ может быть обеспокоен ростом процентных ставок, который увеличит стоимость этого кредита, или столкнется с кредитором, который неохотно будет предоставлять больше кредита, в то время как у компании есть этот риск переменной ставки.

      Предположим, что XYZ создает своп с компанией QRS, которая готова обменять причитающиеся платежи по ссуде с переменной ставкой на платежи по ссуде с фиксированной ставкой в ​​размере 7%. Это означает, что XYZ будет платить 7% QRS по своей основной сумме в 1 000 000 долларов, а QRS будет платить XYZ 6% по той же основной сумме. В начале обмена XYZ просто заплатит QRS разницу в 1% между двумя ставками обмена.

      Если процентные ставки упадут так, что переменная ставка по исходной ссуде теперь составляет 5%, компания XYZ должна будет выплатить компании QRS разницу в 2% по ссуде.Если процентные ставки вырастут до 8%, то QRS придется выплатить XYZ разницу в 1% между двумя ставками свопа. Независимо от того, как меняются процентные ставки, своп достиг первоначальной цели XYZ – превратить ссуду с переменной ставкой в ​​ссуду с фиксированной ставкой.

      Свопы также могут быть созданы для риска обменного курса валют или риска невыполнения обязательств по ссуде или денежных потоков от другой коммерческой деятельности. Свопы, связанные с денежными потоками и потенциальными дефолтами по ипотечным облигациям, являются чрезвычайно популярным видом производных финансовых инструментов, хотя и слишком популярными.В прошлом. Именно риск контрагента подобных свопов в конечном итоге перерос в кредитный кризис 2008 года.

      Опции

      Опционный контракт похож на фьючерсный контракт в том смысле, что это соглашение между двумя сторонами о покупке или продаже актива в заранее установленную дату в будущем по определенной цене. Ключевое различие между опционами и фьючерсами заключается в том, что при наличии опциона покупатель не обязан выполнять свое соглашение о покупке или продаже. Это только возможность, а не обязательство – фьючерсы – это обязательства.Как и в случае с фьючерсами, опционы могут использоваться для хеджирования или спекуляции на цене базового актива.

      Представьте, что инвестор владеет 100 акциями стоимостью 50 долларов за акцию, которые, по его мнению, в будущем вырастут. Однако этот инвестор обеспокоен потенциальными рисками и решает застраховать свою позицию опционом. Инвестор может купить опцион пут, который дает ему право продать 100 акций базовой акции по 50 долларов за акцию (известная как цена исполнения) до определенного дня в будущем, известного как дата истечения срока.

      Предположим, что цена акции упадет до 40 долларов за акцию к истечению срока, и покупатель опциона на продажу решает исполнить свой опцион и продать акции по первоначальной цене исполнения 50 долларов за акцию. Если покупка опциона пут стоила инвестору 200 долларов, то они потеряли стоимость опциона только потому, что цена исполнения была равна цене акции, когда они первоначально купили опцион. Подобная стратегия называется защитным путом, потому что она хеджирует риск падения акций.

      В качестве альтернативы предположим, что инвестор не владеет акциями, стоимость которых в настоящее время составляет 50 долларов за акцию. Однако они считают, что в течение следующего месяца акции вырастут в цене. Этот инвестор может купить опцион колл, который дает ему право купить акции за 50 долларов до или по истечении срока. Предположим, что этот опцион колл стоил 200 долларов, а акция выросла до 60 долларов до истечения срока. Покупатель колл теперь может исполнить свой опцион и купить акцию стоимостью 60 долларов за акцию по цене исполнения 50 долларов, что представляет собой начальную прибыль в размере 10 долларов за акцию.Опцион колл представляет собой 100 акций, поэтому реальная прибыль составляет 1000 долларов за вычетом стоимости опциона – премии – и любых комиссионных комиссионных брокеров.

      В обоих примерах продавцы опционов «пут» и «колл» обязаны выполнить свою часть контракта, если покупатель опциона «колл» или «пут» решит исполнить контракт. Однако, если цена акции выше цены исполнения на момент истечения срока, пут будет бесполезен, и продавец – продавец опциона – получит право удерживать премию по истечении срока действия опциона.Если цена акции ниже страйк-цены на момент истечения срока, колл будет бесполезным, и продавец колл сохранит премию. Некоторые опционы могут быть исполнены до истечения срока их действия. Они известны как варианты в американском стиле, но их использование и раннее применение редки.

      Преимущества производных инструментов

      Как показывают приведенные выше примеры, деривативы могут быть полезным инструментом как для бизнеса, так и для инвесторов. Они позволяют зафиксировать цены, застраховаться от неблагоприятных колебаний ставок и снизить риски – часто за небольшую плату.Кроме того, деривативы часто можно купить с маржой, то есть за счет заемных средств, что делает их еще дешевле.

      Обратная сторона деривативов

      С другой стороны, производные финансовые инструменты трудно оценить, потому что они основаны на цене другого актива. Риски для внебиржевых деривативов включают риски контрагента, которые также трудно предсказать или оценить. Большинство производных финансовых инструментов также чувствительны к изменениям времени до истечения срока, стоимости владения базовым активом и процентных ставок.Эти переменные затрудняют точное согласование стоимости производного инструмента с базовым активом.

      Плюсы
      • Блокировка цен

      • Хеджирование от риска

      • Можно использовать

      • Диверсифицировать портфель

      Кроме того, поскольку производный инструмент сам по себе не имеет внутренней стоимости – его стоимость определяется только базовым активом, – он уязвим для настроений рынка и рыночного риска. Факторы спроса и предложения могут вызывать рост и падение цены производного инструмента и его ликвидности независимо от того, что происходит с ценой базового актива.

      Наконец, деривативы обычно являются инструментами с кредитным плечом, и использование кредитного плеча сокращает их в обоих направлениях. Хотя это может увеличить норму прибыли, это также приводит к более быстрому росту убытков.

      Пример деривативов в реальном мире

      Многие производные инструменты используют заемные средства. Это означает, что требуется небольшая сумма капитала, чтобы иметь долю в большой стоимости базового актива.

      Например, инвестор, ожидающий роста стоимости индекса S&P 500, может купить фьючерсный контракт, основанный на этом уважаемом индексе акций крупнейшей U.С. публичные компании. Условная стоимость фьючерсного контракта на S&P 500 составляет 250 000 долларов.

      Часто задаваемые вопросы

      Что такое производные?

      Производные финансовые инструменты – это ценные бумаги, стоимость которых зависит от базового актива или «получена из него». Например, фьючерсный контракт на нефть – это разновидность производного финансового инструмента, стоимость которого основана на рыночной цене на нефть. Деривативы становятся все более популярными в последние десятилетия, а общая стоимость непогашенных деривативов в настоящее время оценивается более чем в 600 триллионов долларов.

      Какие примеры деривативов?

      Распространенные примеры производных инструментов включают фьючерсные контракты, опционные контракты и свопы на дефолт по кредиту. Помимо этого, существует огромное количество контрактов с производными финансовыми инструментами, предназначенных для удовлетворения потребностей широкого круга контрагентов. Фактически, поскольку многие деривативы торгуются на внебиржевом рынке (OTC), в принципе их можно бесконечно настраивать.

      Каковы основные преимущества и риски деривативов?

      Деривативы могут быть очень удобным способом достижения финансовых целей.Например, компания, которая хочет застраховаться от воздействия сырьевых товаров, может сделать это, покупая или продавая производные энергетические инструменты, такие как фьючерсы на сырую нефть. Точно так же компания может хеджировать свой валютный риск, покупая валютные форвардные контракты. Деривативы также могут помочь инвесторам использовать свои позиции, например, покупая акции через опционы на акции, а не акции. Основные недостатки деривативов включают риск контрагента, риски, присущие заемным средствам, и тот факт, что сложные сети деривативных контрактов могут привести к системным рискам.{[\ kappa]} \), чтобы быть матрицей размера \ (n_ \ kappa \ times m_ \ kappa \), затем из алгоритма в разд. 3 у нас есть \ ({m_ \ kappa \ atopwithdelims () n_ \ kappa} \) потенциальные системы индекса 1 на стадии \ (\ kappa +1 \). Таким образом, потенциальное количество систем с индексом 1, которые можно получить с помощью этого метода, может быть очень большим для практических примеров. Мы хотели бы использовать структурный анализ разд. 2 для информирования о нашем выборе DD и, таким образом, ограничения потенциального количества систем, рассматриваемых на каждом этапе. См. Раздел. 5 для других способов достижения численного ускорения.{(d_j)}} \ right) = \ mathbf {J}. \ end {align} $$

      (11)

      Переход к следующему этапу в DD путем уменьшения порядка дифференцирования на 1 эквивалентен уменьшению вектора смещения \ (\ mathbf {c} \) на 1 (и, следовательно, уменьшению \ (\ mathbf {d} \) на 1 также). Поэтому на этапе 1 в DD мы будем рассматривать уравнения, используемые на этапе \ (- 1 \) SA, поскольку SA увеличивает порядок дифференцирования на единицу на каждом этапе. Это приводит нас к следующему наблюдению.{[\ kappa]} \)), для каждой стадии k между \ (k _ {\ text {min}} \) и 0.

      Проба

      Мы уже показали, что на этапе 0 оба метода используют одни и те же уравнения. В DD мы удалим все уравнения, такие что \ (c_i = 0 \). Затем мы опускаем одно различие и повторяем. Следовательно, мы удаляем на этапе 1 такие уравнения, что \ (c_i-1 = 0 \), и по индукции на этапе \ (\ kappa \) уравнения такие, что \ (c_i- \ kappa = 0 \), где \ ( \ kappa \) – номер стадии.{(l)} (t) \). Тогда для \ (n \ times n \) DAE у нас есть набор индексов:

      $$ \ begin {align} \ mathcal {J} = \ {(j, l) \ mid j = 1, \ dots, n ; l = 0,1, \ dots \}, \ end {align} $$

      аналогично для уравнений, которые мы используем набор:

      $$ \ begin {align} \ mathcal {I} = \ {(i, l ) \ mid я = 1, \ точки, п; l = 0,1, \ точки \}. \ end {align} $$

      Это дает нам обозначение для переменных, используемых на каждом этапе в SA, а именно:

      $$ \ begin {align} I_k & = \ {(i, l) \ in \ mathcal {I} \ mid l = k + c_i \} \\ J_k & = \ {(j, l) \ in \ mathcal {J} \ mid l = k + d_j \} \ end {align} $$

      , где смещения считаются каноническими, если не указано иное.На каждом этапе SA мы имеем:

      $$ \ begin {align} m_k = \ mid \ mathcal {I} _k \ mid = \ mid \ {j \ mid d_j + k \ ge 0 \} \ mid, \ quad n_k = \ mid \ mathcal {J} _k \ mid = \ mid \ {i \ mid c_i + k \ ge 0 \} \ mid. \ end {align} $$

      Мы пишем \ (f_ {I_k} \) для обозначения набора уравнений, используемых на этапе k в SA, и \ (f_ {I _ {\ le k}} \) для обозначения набор уравнений, используемый между k stage \ (k _ {\ text {min}} \) и k —ie \ (f_ {I_ {k _ {\ text {min}}}} \ cup f_ {I_ {k _ {\ text {min}} + 1}} \ cup \ cdots \ cup f_ {I_ {k}} \), аналогично для переменных.

      Опять же, из [18] мы видим, что системный якобиан, используемый на этапе k в SA, задается следующим образом:

      $$ \ begin {align} \ mathbf {J} _k = \ frac {\ partial f_ {I_k }} {\ partial x_ {J_k}}. \ end {align} $$

      (12)

      Вспомните примечание в конце разд. 3.2, что дает нам следующую лемму:

      Лемма 3

      В DD, если на этапе \ (\ kappa \), \ (d_j = \ kappa +1 \), то столбец j не может находиться в \ (G ^ {[\ kappa +1]} \).{[-k]} \), поскольку они будут столбцами структурных нулей. \(\квадратный \)

      Таким образом, столбцы, представляющие недифференцированные переменные, не могут быть выбраны в качестве DD, как и следовало ожидать.

      Пример 6

      Рассмотрим пример 5 и вспомним систему с индексом 1, данную в формуле. (10). Сравните это с результатами SA в таблице 2, переменные, которые стали DD, отмечены штрихом. В примере 5 мы превращаем подмножество переменных, найденных на этапе k в SA, в DD на этапе \ (- k + 1 = \ kappa +1 \) в схеме DD, используя одни и те же уравнения в обоих случаях, см. Таблицу 2.

      Таблица 2 Результаты SA для линейной DAE (9) – также показаны DD на простое число

      Общее количество введенных DD будет равно \ (\ sum _ic_i \), поскольку это общее количество введенных новых уравнений, и мы идентифицируем один DD с каждым новым уравнением. На каждом этапе переменные, которые создали DD, представляют собой подмножество переменных, которые создали DD на предыдущем этапе (обязательно за исключением переменных с \ (d_j = \ kappa +1 \)), размером \ (m_ {k-1} \) , причем каждая переменная дифференцируется на один раз меньше, чем на предыдущем этапе.{[0]} \) завершает доказательство. \(\квадратный \)

      Пример 7

      В [2] авторы вводят DAE для моделирования руки робота. Он переформулирован как SWP (т.е. иметь конечный Val (\ (\ varSigma \))) в [17]. Мы будем использовать формулировку \ (6 \ times 6 \), введенную в [17], и расположим уравнения и переменные так, чтобы мы ясно проиллюстрировали блочно-треугольную структуру задачи. Структурный якобиан, матрица сигнатур и смещения (с HVT, отмеченным \ (\ bullet \) и \ (- \ infty \) записями, оставленными пустыми) для этой DAE, где \ (F_w \) означает \ (\ partial F / \ частичное w \) и так далее:

      Для последующих наблюдений полезно отметить, что приведенная выше матрица сигнатур имеет четыре грубых (также мелких) блока, два размера \ (2 \ times 2 \) и два размера \ (1 \ times 1 \). {[- k + 1]} \) в схеме ДД, т.е.{(d_j)} \) называются структурно необходимыми фиктивными производными.

      Таблица 4 Этапы SA для уравнения. (8)

      Это дает нам следующий улучшенный алгоритм DD, в котором мы можем идентифицировать структурно необходимые DD без вычисления числовых якобианов:

      Для систем с 0 степенями свободы это идентифицирует все DD, как и следовало ожидать. Очевидно, что алгоритм 2 находит все структурно необходимые DD. Применение первой половины алгоритма 2 (поиск структурно необходимых DD) к манипулятору робота дает нам следующую поэтапную схему решения, созданную расширением (еще не выпущенным) кода авторов DAESA [10]:

      Это нужно сравнить с Таблицей 3, чтобы убедиться в результате.

      Пример 8

      Вспомните модифицированную DAE с двойным маятником (8) и соответствующую матрицу сигнатур и смещения на рис. 1. В таблице 4 приведены этапы решения SA для этой задачи. В таблице 4 мы видим, что все производные переменных более высокого порядка, найденные на этапе 4 в DD, будут преобразованы в DD (то есть те, которые используются на этапах SA \ (- 3, -2, -1 \) или этапах DD 1, 2, 3). Теперь рассмотрим различные BTF, показанные на рис. 1. Применение следствия 1 соответствует тому, что мы решаем первый грубый блок как отдельную систему, а затем используем его для решения второго грубого блока.{[\ kappa]} \) на этапе \ (\ kappa \), например на этапе 4 мы остаемся с системой \ (3 \ times 3 \), заданной первым грубым блоком в нашем BTF, как и ожидалось. Отметим, что этот алгоритм для структурно необходимых DD выглядит аналогично решению для DD на основе грубого BTF, см. Разд. 4.4 почему это не совсем так.

      Пример 9

      Этот улучшенный алгоритм DD действительно достигает нашей цели по сокращению общего количества потенциально необходимых систем индекса 1. Рассмотрим снова DAE (8), проработав структурный анализ, мы видим, что на этапе \ (k = -4 \) мы имеем \ (m_k = n_k \).{[\ kappa]} \) матриц на каждом этапе, которые должны обеспечивать ускорение вычислений при проверке числа условий каждого якобиана при выполнении фиктивного поворота, см. разд. 5. Перед тем, как дать алгоритм поиска DD на блоках, мы спрашиваем, делал ли алгоритм 2 для нас что-то похожее на BTF. Мы рассматриваем мелкую блочную декомпозицию (которая сама по себе является BTF внутри грубой BTF) и спрашиваем, можем ли мы дополнительно уменьшить потенциальные варианты выбора индекса 1, учитывая, что \ (m_k \) равняется \ (n_k \) на этапах SA.

      Теорема 5

      Если задана \ (n \ times n \) DAE и существует \ (k \ in \ {k _ {\ text {min}}, \ dots, -1 \} \) такая, что \ (n_k = m_k = \ mu \) для некоторого \ (0 <\ mu

      Проба

      Подобно доказательству леммы 3, одна разбивает матрицу на следующие, а затем отмечает, что верхний правый блок пуст по (1).

      \(\квадратный \)

      Это означает, что у нас не может быть грубой блочной неприводимой DAE с \ (n_k = m_k \), если только \ (k \ ge 0 \) или \ (k м мелких блоков и мы решаем точный блок м , т.е. находимся на глобальной стадии 0. Следовательно, мы обращаем наше внимание на взаимодействие между блоками, а не пытаемся для дальнейшей оптимизации внутри блока.Мы определяем локальные смещения, связанные с произвольной формой блока:

      Определение 3

      ( Блочные локальные смещения ) Мы обозначаем канонические локальные смещения, связанные с произвольной формой блока, если бы ее блоки рассматривались как автономные системы, как \ (\ check {c} _i \) и \ (\ check {d} _j \).

      Учитывая блочную форму \ (\ varSigma \) с L блоками, мы имеем Алгоритм 3.

      В следующем обсуждении мы будем называть DD блокировать необходимые фиктивные производные , если они найдены в конце алгоритма 3, i .е. если их можно найти, используя локальные смещения блочной формы и глобальные канонические смещения. В алгоритме 3 нет явного взаимодействия между блоками (межблочные зависимости учитываются в строках 19 и 20), и поэтому большая часть алгоритма может выполняться параллельно. В наших обсуждениях ниже мы ограничимся размышлениями о грубых и точных BTF, как описано в Разд. 2.3, поскольку они наиболее естественны при использовании метода сигнатурной матрицы.

      Чтобы доказать, что алгоритм 3 дает подходящий выбор DD, нам нужен аналог теоремы из [20], который утверждает, что разница между локальным и глобальным смещениями постоянна в тонкой блочной форме, чтобы иметь место для произвольной блочной формы. Это означало бы, что мы вводим столько DD, сколько дифференцированных уравнений, принимая во внимание взаимодействия между блоками. Однако рассмотрим следующий пример:

      Пример 10

      Рассмотрим DAE со следующей матрицей сигнатур:

      Здесь мы выполняем блочную треугольную форму над крупными блоками.Между грубыми локальными и глобальными смещениями нет постоянной разницы, поэтому возможно, что мы вводим больше DD, чем мы вводим уравнения при использовании алгоритма 3.

      Потенциальная проблема в примере 10 на самом деле вовсе не проблема:

      Теорема 6

      Учитывая BTF \ (\ varSigma \), разница между суммой локальных и глобальных смещений любого блока равна по отношению к \ (\ mathbf {c} \) и \ (\ mathbf {d} \).

      Проба

      Возьмите матрицу сигнатуры \ (n \ times n \) \ (\ varSigma \) и поместите ее HVT на главную диагональ. Пусть S будет любым подмножеством \ (\ {1, \ dots, n \} \) и \ ({\ check {\ mathbf {c}}} \) и \ ({\ check {\ mathbf {d}) }} \) – любые допустимые смещения. Тогда, поскольку \ (\ check {d} _i- \ check {c} _i = \ sigma _ {i, i} \), мы имеем:

      $$ \ begin {align} \ sum _ {i \ in S} \ check {d} _i – \ sum _ {i \ in S} \ check {c} _i = \ sum _ {i \ in S} \ sigma _ {i, i} = \ text {Val} (S) \ конец {выровненный} $$

      , который не зависит от \ ({\ check {\ mathbf {c}}} \) и \ ({\ check {\ mathbf {d}}} \).Итак, для любых других допустимых смещений, например \ (\ mathbf {c} \) и \ (\ mathbf {d} \), мы имеем:

      $$ \ begin {align} \ sum _ {i \ in S} \ check {d} _i – \ sum _ {i \ in S} d_i = \ sum _ {i \ in S} \ check {c} _i – \ sum _ {i \ in S} c_i = \ text {Val} ( S) \ end {align} $$

      \(\квадратный \)

      Это не то, чего можно было ожидать вначале: в нашем алгоритме грубого блока можно было ожидать дифференцировать весь блок несколько раз, чтобы решить следующий блок, эта теорема показывает, что на самом деле вам может потребоваться только дифференцировать некоторые части блока, чтобы сохранить квадратную систему индекса 1, используя строки 19 и 20 в алгоритме 3.

      Теорема 7

      Алгоритм 3 дает подходящий выбор DD, которые иначе можно было бы найти, рассматривая всю систему и исходный алгоритм DD.

      Проба

      Если можно выполнить описанный выше алгоритм, то мы получим блочную форму, диагональные подматрицы которой структурно невырождены. Поскольку грубые локальные смещения каждого блока являются константой, отличной от глобальной, по лемме 1 мы должны иметь неособый якобиан на каждой глобальной стадии DD, если у нас есть неособый якобиан на каждой грубой локальной стадии.Опять же, по лемме 1 мы видим, что допустимый выбор для алгоритма DD на глобальном этапе – это сочетание переменных, найденных на положительных и отрицательных локальных этапах, с использованием времени выполнения каждого блока (если мы рассмотрим добавление переменных и уравнений в конце алгоритма как выполнение локальных этапов \ (0, \ dots, \ max _j {(d_j- \ check {d} _j}) \)). \(\квадратный \)

      Из-за теоремы 5 и примера 7 можно подумать, что при ограничении грубыми блоками Алгоритм 3 – это просто переформулировка алгоритма 2 – рассмотрим DAE с матрицей сигнатур:

      Ищем этапы, где \ (m_k = n_k \) мы не находим структурных DDs.{(4)} \).

      Мы видим, что алгоритм 3, ограниченный грубым BTF, дает действительный набор DD, который можно найти с использованием глобальных смещений и без BTF. Все, что мы сделали, это взяли BTF, поэтому можно было бы предположить, что в целом он генерирует все возможные наборы допустимых DD, которые могут быть получены в глобальном масштабе, как, несомненно, утверждается в исходной статье [7]. Однако подумайте:

      Пример 11

      Рассмотрим DAE с матрицей подписи следующим образом:

      Обратите внимание, что грубый BTF, приведенный выше, такой же, как точный BTF и \ (\ mathbf {c} = \ check {\ mathbf {c}} = \ hat {\ mathbf {c}} \) и \ (\ mathbf {d} = \ check {\ mathbf {d}} = \ hat {\ mathbf {d}} \).Алгоритм 3 выдаст DD для \ (\ ddot {x} _1 \) или \ (\ dot {x} _2 \) из первого блока, то есть алгоритм 3 не может найти набор DD, где ни один из \ (\ ddot {x} _1 \) и \ (\ dot {x} _2 \) выбраны. Однако, если мы выполним исходный алгоритм DD, мы увидим, что существует индексная система, которая использует только производные переменных \ (x_3 \), \ (x_4 \) и \ (x_5 \) в качестве DD. Таким образом, есть системы индекса 1, которые мы можем «пропустить», используя BTF, которые в противном случае были бы доступны, если рассматривать DAE в целом.

      фиктивные производные на основе точных блоков

      Теперь мы покажем, что выбор формы блока напрямую влияет на количество необходимых блоков DD, как определено в разд.4.4. Мы утверждаем, что прекрасный BTF – хороший выбор. Прежде чем привести пример алгоритма 3 с использованием точного BTF, мы задаемся вопросом, существует ли более информативная версия алгоритма 2, основанная на точном BTF. Рассмотрим снова уравнение. (8), его матрицы сигнатур на рис. 1 и этапы в таблице 5. Для этапов \ (- 2 \) и \ (- 1 \) \ (m_k \ ne n_k \), потому что мы должны ввести начальные значения для \ (\ ddot {x} _5 \) и \ (\ dot {x} _5 \). Если бы нам не пришлось этого делать, у нас снова была бы квадратная система, на этот раз с 4 уравнениями – исходный грубый блок, содержащий уравнения 1, 2, 3 и переменные 1, 2, 3, и тонкий блок, содержащий уравнение 6 и переменную 4 .Отметим, что переменная 5 фактически не появляется в схеме решения DD до этапа 0, потому что у нее нет связанного уравнения. Поэтому мы можем игнорировать любые переменные, которые должны быть указаны как IV, и оставим следующее определение

      Определение 4

      Мелкие блочные локальные номера уравнений и переменных записываются как:

      $$ \ begin {выровнено} \ widehat {m} _ {k, l} = \ biggl | \ {j \ text {in block} l \ mid \ widehat {d_j} + k \ ge 0 \} \ biggr |, \ quad \ widehat {n} _ {k, l} = \ biggl | \ {i \ text {in block} l \ mid \ widehat {c_i} + k \ ge 0 \} \ biggr | \ end {align} $$

      , где k рассматривается как локальный этап SA, связанный с точным блоком.

      В результате мы получаем улучшенный аналог алгоритма 2 на основе точных блоков. Для каждого точного блока l выполните следующие действия (где \ (\ widehat {k} _ {\ text {min}, l} \) – это \ (- \ max _jd_j \) таким образом, что j находится в блоке l ):

      Зная это, это побуждает нас продолжать использовать точный BTF для поиска DD, поскольку у нас есть лучший набор структурных DD (мы будем называть такие DD структурно необходимыми DD), чем было ранее найдено с использованием всей матрицы сигнатур.

      Пример 12

      Рассмотрим снова DAE в примере 4. Мы выполняем алгоритм 3 для этой DAE, чтобы проиллюстрировать преимущества точного BTF. Локальные смещения говорят нам, что у нас должны быть DD, показанные в таблице 6, найденные путем сравнения смещений по строкам 19 и 20 алгоритма.

      Таблица 6 DD для мелких блоков, например (8)

      Единственный блок, в котором мы должны выбрать DD, – это блок 4 (эквивалент простого маятника).2 & = 0 \ end {выравнивание} \ right \} \ end {выравнивание} $$

      Выполнение алгоритма DD дает две возможные системы индекса 1, где выбор DD приведен в таблице 7. Мы можем найти 14 блоков необходимые DD без необходимости вычислять числовой якобиан и с самого начала уменьшили размер нашей задачи вдвое.

      Таблица 7 Возможные варианты DD из (8)

      Ниже приводится краткий анализ сложности. Предположим, DAE раскладывается на L мелких блоков, помеченных индексом l .3 \ right) \ end {align} $$

      , где \ (\ widehat {c} _i \) во втором суммировании берется из блока l . Предполагая, что система распадается на относительно небольшие мелкие блоки, то есть \ (\ widehat {n} \ ll n \) и некоторые \ (\ widehat {c} _i

      Важно отметить, что значительное сокращение потенциальных DD и размера задачи делает поворот манекена менее проблематичным, поскольку нам придется рассматривать меньшие системы уравнений с уменьшенным числом возможных переменных для выбора, см. Разд.5 для более подробной информации.

      Рассмотрим еще раз Пример 11, нам напоминают, что есть системы с индексом 1, которые мы можем «пропустить», используя точный BTF, который в противном случае был бы доступен при рассмотрении DAE в целом. Хотя это кажется довольно серьезным упущением в отношении нашего метода, этот потенциальный «надзор» никогда не сделает неразрешимую иначе решаемую (с помощью DD) DAE:

      Теорема 8

      Если DAE разрешима, то алгоритм 3, ограниченный точным BTF, всегда сможет выбрать допустимый набор DD.

    Оставить комментарий